BIOL 2124 Student Bank Questions - Part 2

¡Supera tus tareas y exámenes ahora con Quizwiz!

317. You are at the library studying really late for your physiology exam. At midnight, you decide it is time to go home. As you walk outside to your car, you hear someone behind you. You turn around to see a shadow creeping closer and closer towards you. You panic and begin running. Which of the following is not occurring in your body? A. Your skeletal muscle is stimulated to contract more. This is done by releasing more calcium into the sarcoplasm from the sarcoplasmic reticulum to be used for excitation contraction coupling. B. Your heart rate will increase in order to increase blood flow. C. Your pupils will dilate due to the contraction of radial muscles in the eye so that more light can be let in. D. The beta cells in the pancreas will be inhibited from secreting insulin. E. Bronchodilation will occur in your lungs in order to increase the amount of oxygen you can take in while running away.

(HE WOULD HAVE TO REPHRASE THIS... A AND D COULD BE ANSWERS)

377. Karen's coronary artery gets stabbed by a surgeon on accident. What effects will Karen have? a. She will not be able to get blood to her heart muscle so it will stop working as efficiently. b. She will not be able to get blood to all her systemic organs c. She will not be able to get blood to the brain d. Nothing because her brain and arteries will still receive blood e. Her heart muscle, brain, and organs will all not receive blood.

a

391. Which of the following is false regarding proteins? a. They function as enzymes b. During protein digestion in the small intestine HCl converts pepsinogen to pepsin. Pepsin then breaks down proteins into polypeptides c. When completely digested they are broken down into Amino Acids d. They are absorbed via active transport e. They are made of an Amino group and R group and a Carboxyl group.

b

259. Lydia decided to give blood one day at an American Red Cross blood-drive. After her donation she was given a blood card that stated that Lydia had A+ blood. Which of the following statements is true about Lydia's A+ blood type? a. Antigen B will be found in Lydia's blood located in the blood plasma. b. Leukocytes in Lydia's blood help transport oxygen and carbon dioxide. c. Anti-B antibody will be found in Lydia's blood located in the blood plasma. d. Lydia can donate blood to people with O+ blood type. e. If Lydia donates blood to a person with Rh- antibodies that person will not develop Rh+ antibodies.

c

398. Which of the following situations would be most likely to NOT result in hypoxia? a. sickle cell anemia b. inability of the intestines to properly absorb iron c. an abnormally low level of fast glycolytic fibers in skeletal muscle d. hypersecretion of calcitonin e. prolonged exposure to dust over one's lifetime

c

405. Which of the following statements are correct? A. To be able to move something in or out of a cell using secondary active transport, ATP must be used. B. The electron transport chain produces the most ATP during carbohydrate metabolism. C. Digestion for carbohydrates starts in the mouth from the enzyme, salivary amylase. D. Lipophobic messengers are made on demand, and are not stored. E. Muscarinic Cholinergic Receptors only use a second messenger system.

c

417. Which of the following statements is false regarding the ion potassium? a. Potassium has more movement across a typical cell membrane, due to leak channels being more permeable to potassium than sodium. This then creates a negative resting membrane potential of the cell because more positive ions are moving out of the cell than into it. b. Adding a minimal amount of potassium to the heart's extracellular fluid increases the resting membrane of the cell. This can indirectly cause hyperventilation, which makes the blood more alkaline. c. During the process of hearing, the stereocilia of the ear bend toward the kinocilium, which causes potassium to efflux from the cell. This then allows calcium to flow in and increase action potential frequency. d. When the conductance of potassium is increasing across a typical neuron its voltage gated channels are opening. This happens near +30mV, during the absolute refractory period. e. Adding an excess amount of potassium to the heart's extracellular fluid keeps pacemaker cells from spontaneously depolarizing, which will decrease the force of contraction in the heart to a halt.

c

386. Alice, who is 75 years old and considered severely overweight, had a stroke and was rushed to the hospital. Upon awakening, she discovers that the right side of her body is completely paralyzed. Her doctors explain that her stroke has caused severe brain and neural damage. What area of the brain did the stroke affect, and why did it cause her to become partially paralyzed? a. The stroke caused a blockage to her hypothalamus which ceased the release of neurotransmitters such as epinephrine that allow her brain to send neural signals to her body for movement. b. The stroke caused overstimulation of the motor cortex to release neurons that overwhelm the body due to the sense of danger during stroke. This caused the neural receptors on the motor cortex to become desensitized and down regulate. These receptors cease to respond to stimulation, and signaling to the body's extremities slow down until the receptors completely stop responding. At this point the area of the body is paralyzed. c. The stroke blocked important chemoreceptors for blood pH levels on the pons and medulla. This damage caused the body to not be able to control blood pH, and without a balanced pH blood is not be able to flow throughout the body correctly. Without blood flowing throughout the body the limbs become immobile. d. The stroke caused a blood clot in artery in the right side of Alice's body. This artery supplied neural communication from the heart to the brain, and without blood flow being supplied to the tissues in the brain, these tissues soon die and dead tissue cannot be repaired. The line of communication between the brain and the body has been severed and this area of the body becomes paralyzed. e. The stroke intercepted the neurons that control the communication between the body and the entire left hemisphere of the brain. Without this communication, the brain is not able to signal movement for the body

d

389. Sydney has been feeling down for the past of couple months. She doesn't feel like participating in her normal activities and is withdrawing more and more from her loved ones. Sydney is encouraged to go see a therapist. The psychiatrist who evaluates her prescribes Prozac (fluoxetine hcl). Prozac is an anti-depressant drug and is classified as a Selective Serotonin Reuptake Inhibitor (SSRI). How will Prozac work to elevate Sydney's mood? a. It will increase her production of the hormone serotonin, which results in boosting her mood. b. It will allow her platelets to produce more serotonin, enhancing vasoconstriction. c. It will leave serotonin in the synaptic cleft for a longer period of time due to the medication preventing uptake of the neurotransmitter into the post-synaptic neuron. d. It will allow serotonin to be left in the synaptic cleft for a longer period of time due to the medication hindering uptake of the neurotransmitter back into the pre-synaptic neuron. e. It will enable serotonin to be in the synaptic cleft for a shorter period of time, allowing the neurotransmitter to travel to the other parts of her body faster.

d

395. Mark has a blood disorder known as hyponatremia, which refers to a low level of sodium in the blood. Note that this decrease of sodium in the blood does not reverse the concentration gradient for sodium. How could this disorder affect Mark's body? A. This disorder could decrease contractility of Mark's heart by slowing down the influx of sodium, due to a decrease in the driving force for sodium, during slow depolarization through the funny sodium channels in cardiac contractile cells. B. This disorder could increase the time it would take for Mark's skeletal muscle to relax due to a decrease in the driving force for sodium to move into the cell to let calcium leave the cell via secondary antiport. C. This disorder could decrease Mark's heart rate by slowing down the influx of sodium, due to a decrease in the driving force for sodium, during slow depolarization through the funny sodium in cardiac contractile cells. D. This disorder could increase the time it takes for Mark's smooth muscle to relax due to the decrease in the driving force for sodium to move into the cell to let calcium leave the cell via secondary antiport. E. This disorder could decrease Mark's heart rate by slowing down the influx of sodium, due to a decrease in the driving force for sodium, during slow depolarization through voltage gated sodium channels in autorhythmic cells.

d

399. Heather went to the doctor's office to get a CT scan because she was feeling pain in her lower back around her kidneys. The scan showed that she had a tumor on her kidney and was causing her to release large amounts of Renin. Which of the following is occurring is Heather's body? A. Aldosterone would be secreted in excess causing sodium to be excreted and potassium to be reabsorbed. B. Heather will have frequent urination. C. Heather is very jittery because of the excess epinephrine being secreted by the posterior pituitary. D. Heather's blood pressure is very high due to Angiotensin II causing vasoconstriction of blood vessels. E. The pain Heather is feeling is caused by action potentials going up the afferent axon through the ventral root ganglion to the CNS.

d

416. As Christmas draws near, Mrs. Claus understands that it is a stressful time for Mr. Claus. However, she becomes concerned as she notices his strange behaviors including an insatiable appetite for gingerbread cookies, and that he would barely wear any clothes despite the North Pole's freezing weather. She tells Santa Claus to visit the elf doctor. Upon his visit to the elf doctor, Santa Claus explains that he has been experiencing weight loss, an increased appetite, a lower tolerance for the heat, and increased heart rate. The elf doctor explains to Santa Claus that he may be hypersecreting thyroid hormone (TH) and diagnoses him with hyperthyroidism. Based on this knowledge which of the following is true? A. The elf doctor prescribes antithyroid drugs to Santa in order to increase the synthesis of TH. B. Santa's hypothalamus is secreting less TRH in response to the hypersecretion of TH by the thyroid gland. This is stimulated by short loop negative feedback. C. The excessive amount of TH freely diffuses through the plasma membrane, travels through Santa's blood via a carrier protein, and binds to receptors on the plasma membrane of target cells. D. There is a high number of mitochondria in Santa's cells. E. The Na-I symporter is bringing more sodium and iodine into follicular cells via endocytosis in order to help synthesize more TH.

d

394. What are the three main phases of an action potential (in the correct order) and what causes these phases to take place in a cardiac contractile cell? A. The three main phases of an action potential are depolarization, repolarization, and hyperpolarization; depolarization is caused by sodium influx, repolarization is caused by calcium influx and potassium efflux which causes a plateau until the calcium channels close, and hyperpolarization does not occur B. The three main phases of an action potential are repolarization, depolarization, and hyperpolarization; repolarization is caused by calcium efflux and potassium efflux which causes a plateau until the calcium channels close, depolarization is caused by sodium influx, and hyperpolarization is caused by excess sodium efflux and after depolarization. C. The three main phases of an action potential are depolarization, repolarization, and hyperpolarization; depolarization is caused by sodium influx, repolarization is caused by calcium and potassium efflux, and hyperpolarization does not occur. D. The three main phases of an action potential are repolarization, depolarization, and hyperpolarization; repolarization is caused by calcium influx and potassium efflux which causes a plateau until the calcium channels close, depolarization is causes by sodium influx, and hyperpolarization does not occur. E. The three main phases of an action potential are depolarization, hyperpolarization, and repolarization; depolarization is caused by sodium influx, hyperpolarization is caused by calcium efflux, and repolarization is caused by potassium influx and calcium efflux which causes a plateau until calcium channels close.

a

396. If the mean arterial pressure (MAP) is above normal what would be the mechanism to bring MAP back to normal during a parasympathetic response? A. A preganglionic neuron projects from the lateral horn of the spinal cord and enters the parasympathetic ganglion where its synapses with a postganglionic fiber which then releases Ach on muscarinic receptors which are G-protein coupled this then will lower heart rate. B. A preganglionic fiber from the sacral region travel to the spinal nerves C. A preganglionic fiber release norepinephrine or epinephrine to terminal ganglion in the SA and AV nodes of the heart, slowing the heart rate. D. A preganglionic fiber releases ACh this stimulates the postganglionic neuron to release norepinephrine and epinephrine from varicosities. E. A preganglionic neuron projects from the lateral horn of the spinal cord and enters the terminal ganglion where its synapses and release Ach to nicotinic cholinergic receptors activating the cAMP second messenger system which will bring heart rate back to normal. This will lower MAP to normal.

a

400. It was, sadly, the very last day of lab. Being Dr. Bright's most favorite students, he threw the class a party, which included a great big delicious chocolate cake! Meanwhile........Dr. Ketchum was up in her office admiring a nice specimen of a cockroach. Oh no! She dropped the cockroach into a beaker of "Super-Secret Radioactive Solution!" It started growing...and growing...and growing! Now it was gigantic, bigger that the desk, and--it was hungry! Smelling the chocolate cake, the Monstrosity raced down the stairs and burst into the class room. "AAAAHHHH!" everyone screamed. Some students jumped out the window; some were frozen by fright; others started shouting and throwing things at the Monster Cockroach! Haley screamed "Don't let that dirty roach get the cake!" Dr. Bright threw himself between the cake and the Monster and began smacking it with his cap. Gabby and Alyx pummeled it with rulers. Finally Dr. Bright dispatched the Giant Cockroach. "BRAIN FREEZE!" he shouted as he knocked it over the head with a gallon of ice cream. The people in the above scenario exhibit the stress response. Which of the following describes what might be happening physiologically: A. The hypothalamus signals the Adrenal Medulla. The Adrenal Medulla is part of the sympathetic nervous system in the autonomic nervous system. This releases epinephrine. Epinephrine binds to the B1 adrenergic receptors in the SA node on the atria, increasing more action potentials which increases heart rate and increases force of contraction. B. The hypothalamus signals the Adrenal Cortex. The Adrenal Medulla is part of the parasympathetic nervous system in the autonomic nervous system. This releases epinephrine. Epinephrine binds to the B1 adrenergic receptors in the SA node on the atria, causing more action potentials which increases heart rate and increases force of contraction. C. The hypothalamus signals the Adrenal Medulla. The Adrenal Medulla is part of the sympathetic nervous system in the autonomic nervous system. This releases epinephrine. Epinephrine binds to the nicotinic cholinergic receptors in the SA node on the atria, causing more action potentials which increases heart rate and increases force of contractions. D. The hypothalamus signals the Adrenal Cortex. The Adrenal Medulla is part of the sympathetic nervous system in the central nervous system. This releases epinephrine. Epinephrine binds to the B1 adrenergic receptors in the SA node on the atria, causing more action potentials which increases heart rate and decreases force of contraction. E. The hypothalamus signals the Adrenal Medulla. The Adrenal Medulla is part of the parasympathetic nervous system in the autonomic nervous system. This releases epinephrine. Epinephrine binds to the B1 adrenergic receptors in the SA node on the atria, causing more action potentials which increasing heart rate and decreases force of contraction.

a

406. Logan is a long-distance runner. It takes her at least 30 minutes before she starts to feel even slightly fatigued. However, she has a hard time lifting 10 pound weights. Logan's high endurance levels are caused by her high number of slow oxidative fibers. Why do you think capillaries play a large role in this muscle fiber type? Is there an increase or decrease in venous return while running for long periods of time? A. The capillaries allow for gas exchange and deliver lots of oxygen to her muscles while she is running so her rate of fatigue is low; increase B. The capillaries deliver lots of oxygen to her muscles while she is running so her rate of fatigue is high; decrease C. The capillaries play an important role in fast glycolytic fibers, not slow oxidative fibers; decrease D. The capillaries allow for aerobic respiration to occur which allows for more ATP production which causes her rate of fatigue to be high; increase E. The capillaries play no role in this process; remain unchanged

a

411. Olivia has been bitten by an unknown spider and is now being rushed to the hospital. The doctors immediately discover that she is experiencing paralysis and the slowing of her heart rate. The doctors want to know the best reason for the occurring symptoms and how to quickly ease the patient. What would be your best explanation for the doctors? a. The poison of the spider is acting as an antagonist and slowing the rate of pacemaker and action potentials of the heart. With lower stroke volume, the tissues receive less oxygen and therefore ATP cannot be produced causing paralysis. The doctors should inject epinephrine to speed the heart rate up and allow the body to receive more oxygen. b. The poison of the spider is acting as an agonist and slowing the rate of the pacemaker and action potentials of the heart. With lower stroke volume, the tissues receive less oxygen and therefore ATP cannot be produced causing paralysis. The doctors should inject epinephrine to speed the heart rate up and allow the body to receive more oxygen. c. The poison of the spider is acting as an antagonist and slowing the rate of pacemaker and action potentials of the heart. With higher stroke volume, the tissues receive more oxygen and therefore ATP can be produced causing paralysis. The doctors should inject ACh to speed the heart rate up and allow the body to receive more oxygen. d. The poison of the spider is acting as an agonist and slowing the rate of pacemaker and action potentials of the heart. With lower stroke volume, the tissues receive less oxygen and therefore ATP can be produced causing paralysis. The doctors should inject ACh to speed the heart rate up and allow the body to receive more oxygen. e. The poison of the spider is acting as an antagonist and slowing the rate of pacemaker and action potentials of the heart. With lower stroke volume, the tissues receive less oxygen and therefore ATP cannot be produced causing paralysis. There would therefore be no possible cure.

a

413. Hue M. Fizz has been having problems with persistent coughing and shortness of breath. Hue's doctor diagnosed him with cystic fibrosis. What are some of the observations Hue's doctor may have made to lead him to this diagnosis? A. An obstructive disease presenting symptoms including increased time to exhale. Also, a thick mucus membrane in the lungs caused by blocked solute transport across the lung membrane. B. An obstructive disease presenting symptoms including a low IRV. Also, a thick mucus membrane in the lungs caused by blocked solute transport into the lung membrane. C. A restrictive disease presenting symptoms including a low IRV. Also, thinned mucus in the lungs caused by increased solute transport across the lung membrane. D. An obstructive disease presenting symptoms including increased time to exhale. Also, a thick mucus membrane in the lungs caused by increased solute transport across the lung membrane. E. A restrictive disease presenting symptoms including decreased time to exhale. Also, a thin mucus membrane in the lungs caused by blocked solute transport across the lung membrane.

a

384. Which of the following statements are true regarding the cardiac cycle? a. Atrial diastole and systole occur after ventricular diastole and systole b. The blood is pumped through the atria to the ventricles and then it goes into the aorta c. The AV node is the first stop when it comes to the conduction system d. Norepinephrine and epinephrine bind to a cholergenic receptor e. The P wave on an EKG represents the ventricular depolarization.

b

387. In the 4th century BC, the philosopher Socrates was sentenced to death for allegedly corrupting the youth and being impious. His execution consisted of drinking a poison derived from the hemlock plant, which contains a chemical called coliine. This chemical works by blocking the cholinergic receptors at the neuromuscular junction. Based on this information, what would have been the most likely cause of Socrates' death? a. The chemical blocked the nicotinic cholinergic receptors on the myocardium of the heart, causing them to stop contracting and preventing blood and oxygen from getting to the brain. b. The chemical blocked the nicotinic cholinergic receptors on the muscle fibers of the diaphragm, effectively paralyzing it and preventing Socrates from respiring. This eventually resulted in Socrates being unable to get oxygen to the brain and subsequent death of neurons. c. The chemical blocked the binding of acetylcholine to the muscarinic cholinergic receptors on the diaphragm, causing it to contract and preventing Socrates from inhaling oxygen. This resulted in Oxygen not being delivered to the brain and subsequent death of neurons. d. The chemical bound to the nicotinic cholinergic receptors of the myocardium, causing Socrates to get a parasympathetic response, which caused his heart rate to decrease and not enough blood to get to his brain. e. The chemical bound to the nicotinic cholinergic receptors of the smooth muscle in the bronchial tubes, causing them to constrict and preventing Socrates from being able to breath in oxygen. This prevented oxygen from getting to the brain, resulting in brain death.

b

388. Which of these statements is true regarding posterior pituitary hormones? a. They are stored in the posterior pituitary gland located in the brain where they are synthesized and released b. When vasopressin is released it targets the kidneys and is used for water balance within the body. c. When oxytocin binds to receptors on the bladder it triggers the parasympathetic nervous system to allow you to go to the restroom by contracting the smooth muscle in your bladder d. ADH and oxytocin are considered neurohormones because of the muscarinic receptors that they bond to on the motor end plates in the hypothalamus gland e. Antidiuretic hormone is classified as an amino peptide and is stored in the posterior pituitary

b

390. John came into the emergency room wit lightheadedness and fatigue. He has a past history of emphysema, and continues to smoke daily. What negative effects would emphysema have on the body to cause his side effects? a. Emphysema causes more blood flow through the capillaries so the heart would not be working as much causing fatigue b. Less free airspace in the lungs causes less gas exchange in the capillaries and more stress on the heart causing the lightheadedness and fatigue c. More free airspace in the lungs causes less gas exchange in the capillaries and more stress on the heart causing the lightheadedness and fatigue d. The decreased alveolar pressure would cause more gas exchange to happen and a buildup of O2 would occur causing the lightheadedness and the added stress on the heart causing fatigue e. Emphysema does not cause problems to gas exchange in the lungs which in turn does not cause heart problems either

b

402. Francisco Javier was reminiscing about the time he took some classes of anatomy and physiology and learned a thing or two about his kidneys. He only remembers certain things such as the kidneys filter blood and that the word "adrenal" means "above the kidney." However, he did not find this information helpful in figuring out why he was having trouble urinating. Based on the following scenarios, can you help Francisco figure out what is wrong? A. The lipophilic peptide, ADH (which can diffuse freely through the membrane) is being hyposecreted by the posterior pituitary, thus reabsorbing less water and reducing the volume of urine production B. The lipophobic peptide (which requires a transmembrane protein to get through the membrane), ADH is being hypersecreted by the anterior pituitary, thus reabsorbing more water and reducing the volume of urine production. C. The lipophilic amine (which can freely diffuse through the membrane), vasopressin is being hypersecreted by the anterior pituitary, thus reabsorbing more water and reducing the volume of urine production D. The hydrophilic steroid (which requires a transmembrane protein to get through the membrane), cortisol is being hypersecreted by the adrenal cortex causing an elevation of stress levels, thus reabsorbing more water and reducing the volume of urine production E. The hydrophobic peptide (which can diffuse freely through the membrane), Aldosterone is being hypersecreted by the adrenal cortex thus reabsorbing less water and reducing the volume of urine production

b

407. Sana Lee is a 24 year old female who is pregnant with her first child. When she went for her monthly checkup, the doctor told her that they were going to give her Rh immunoglobulin (RhIg) to protect the baby because of the different blood types that they both had. What was the mother's blood type and what was the child's blood type and why was this a problem? A few months later, she starts to go into labor. However, the doctor tells the family that a specific hormone level has caused her contractions to be extremely slow and it is causing her to have an exceedingly long labor which is not healthy for the baby or the mother. Her family and friends also confessed to the doctor that they had noticed how she had been sitting around being moody every single day and seemed to be depressed throughout the pregnancy. If you were that doctor, what hormone would you have said caused this to occur and what would the levels of this hormone be? A. The mother's blood type was Rh-positive and the baby's blood type was Rh-negative. This would cause the mother to produce Rh antibodies as an immune response. This is a problem because it will destroy the baby's blood cells. Since there was depression seen, the mother most likely had low levels of dopamine. She also most likely had low levels of oxytocin which is the hormone that uses the positive feedback system to contract the uterus so that the baby can be born. B. The mother's blood type was Rh-negative and the baby's blood type was Rh-positive. This would cause the mother to produce Rh antibodies as an immune response. This is a problem because it will destroy the baby's blood cells. Since there was depression seen, the mother most likely had low levels of dopamine. She also most likely had low levels of oxytocin which is the hormone that uses the positive feedback system to contract the uterus so that the baby can be born. C. The mother's blood type was Rh-positive and the baby's blood type was Rh-negative. This would cause the mother to produce Rh antibodies as an immune response. This is a problem because it will destroy the baby's blood cells. Since there was depression seen, the mother most likely had low levels of dopamine but she would have had high levels of epinephrine which causes a parasympathetic response and stops the uterus from contracting. D. The mother's blood type was Rh-negative and the baby's blood type was Rh-positive. This would cause the mother to produce Rh antibodies as an immune response. This is a problem because it will destroy the baby's blood cells. Since there was depression seen, the mother most likely had high levels of dopamine and high levels of oxytocin which is the hormone that uses the positive feedback system to contract the uterus so that the baby can be born. E. The mother's blood type was Rh-positive and the baby's blood type was also Rh-positive. This would cause the mother to produce Rh antibodies as an immune response. This is a problem because it will destroy the baby's blood cells. Since there was depression seen, the mother most likely had high levels of dopamine but she would have had low levels of epinephrine which causes a parasympathetic response and stops the uterus from contracting.

b

409. Sade complained of increased numbness, pain and swelling in parts of her legs, arms and pelvis, shortage of breathe, fatigue, lack of energy, headache and dizziness. Her doctor diagnosed her of having chronic atherosclerosis (plaque build up inside the arteries) in her some for her major arteries. Which of the following is likely to occur? A. This causes an increased blood flow in the arteries supplying her skeletal thereby increasing the amount of calcium in the cytoplasm causing a decrease in the force of contraction in the skeletal muscles. B. This causes a reduced blood flow as well as narrowing and reduction of the arteries that supplies the skeletal muscles. C. This causes a reduced blood flow to part of her lungs causing a reduction in surface area for gas exchange. D. This causes an increase blood flow to part of her lungs causing build up of fluid between her pulmonary capillaries and alveoli. E. This causes a reduced blood flow in the veins that supply her skeletal muscle thereby increasing the pressure in the veins supplying her skeletal muscle.

b

420. It is known that Carbon dioxide is transported by three major mechanisms. Which of the following options is correct about these mechanisms. a. CO2 is transported in hemoglobin, in Red Blood Cells, and in White Blood Cells. b. CO2 is transported in erythrocytes, in the plasma, and after H2CO3- formation. c. CO2 is transported in blood vessels, as HCO3-, in antibodies. d. CO2 is transported in erythrocytes, in the plasma, and after H2CO2 formation. e. CO2 is transported in antibodies, hemoglobin, and plasma.

b

421. Dr. Ketchum is taking her forensic entomology class out to the field for the very first time. One of her students, Adrenna Lynn Rush, was not paying attention to her surroundings and was bitten by a strange bug. This bug injects a toxin that acts as a sympathomimetic drug. Which of the following statements is TRUE regarding what might happen to Adrenna as a result of this bite? A. This drug will increase stroke volume via intrinsic control due to an increased heart rate and an increased force of contraction. B. This drug could bind to Beta-2 adrenergic receptors, activating the cAMP second messenger system; this results in vasodilation which decreases TPR, decreases MAP, and increases blood flow to cardiac and skeletal muscle. C. This drug will increase the amount of calcium binding to tropomyosin in cardiac muscle; this increases the number of cross bridges formed, therefore increasing force of contraction. D. This drug mimics the fight-or-flight response and therefore initiates a fast response via adrenergic receptors. E. This drug augments voltage-gated funny channels, voltage-gated T-type calcium channels, and voltage-gated potassium channels; this results in a higher frequency of action potentials and therefore an increased heart rate.

b

408. While walking through a dark alley you hear noises coming from being you. Your body secretes ______ allowing you to take off running as fast as you can to try to get away from whoever is behind you (if there is anyone). With the secretion of this hormone your heart rate would ______ and your blood pressure would ______. A. Melatonin; increase; decrease B. Epinephrine; increase; increase C. Serotonin; decrease; decrease D. Testosterone; decrease; increase E. Norepinephrine; increase; increase

b and e

401. Bob was beheaded by a guillotine. Which of the following is true regarding the event? A. His heart will stop immediately after. B. Copious amount of calcium will be released into the ECF. C. A portion of his parasympathetic autonomic nervous system is destroyed. D. His body is still able to produce thyroid hormones from the release of TRH. E. His only reconditioning organ is chopped off.

c

410. With the increasing popularity of electronic cigarettes, a spike has been observed in the number of cases of nicotine poisoning. The symptoms of nicotine poisoning occur in two phases, first a phase of stimulation followed by a phase of depression. During the stimulation phase, many patients experience increased heart rates and blood pressures followed by rapid decreases in both heart rate and blood pressure. What is the cause of these changes? A. Since nicotine is an acetylcholine agonist that effects the body's nicotinic cholinergic receptors, nicotine binds to these slow sodium channels that use G-proteins to depolarize the smooth muscle in the heart and veins. B. Since nicotine is an acetylcholine agonist that effects the body's nicotinic cholinergic receptors, nicotine binds to the nicotinic cholinergic receptors at the ganglionic synapse, sending parasympathetic and sympathetic messages to the muscles of the heart and veins. C. Since nicotine is an acetylcholine agonist that effects the body's nicotinic cholinergic receptors, nicotine binds to the nicotinic cholinergic receptors at the ganglionic synapse, sending sympathetic messages to the muscles of the heart and veins, causing the stimulatory reaction. Once the muscles down regulate to these impulses, the depressive effects of the poisoning become apparent. D. Since nicotine is an acetylcholine agonist that effects the body's nicotinic cholinergic receptors, nicotine binds to the nicotinic cholinergic receptors of the parasympathetic nervous system in the heart and veins. E. Since nicotine is an acetylcholine antagonist that effects the body's nicotinic cholinergic receptors, nicotine binds to the nicotinic cholinergic receptors of the parasympathetic nervous system in the heart and veins, which stops these messages from reaching the heart and veins. As such, the uncontested messages from the sympathetic nervous system cause the stimulatory response before the muscles down-regulate causing the depressed response.

c

385. Which of the following is NOT true about the enteric nervous system: a. The enteric nervous system utilizes muscarinic cholinergic receptors to control visceral organs. b. The enteric nervous system ONLY innervates involuntary muscles. c. Nerves of the enteric nervous system synapse in the spinal ganglion. d. The enteric nervous system is least active during physical activity. e. If enteric nervous innervation is interrupted, the concentration of limit dextrins in the gut would increase.

c

392. Polly Mae is a new nurse at the Norman HealthPlex Hospital, she is consistently moving around, with hardly any breaks, and helping patients. Lately, while she is helping Billy Jones during her night shift, she has been feeling dizzy and weak. Billy Jones suggest she go to Doctor Fred's. When she meets Doctor Fred, he takes her heart rate. Dr. Fred then finds that Polly has an increased heart rate. What would most likely cause Polly Mae's increase heart rate and what branch of the autonomic nervous system would be in control. A. Parasympathetic nervous system, the autonomic nervous system releases calcium into the SA node, which causes an increase of heart rate. B. Sympathetic nervous system, the sympathetic nervous system releases acetylcholine in the SA node causing and increase within the pacemaker potential, resulting in an increase heart rate. C. Sympathetic nervous system, due to the sympathetic nervous system activation in the SA node, norepinephrine is released causing an increase within the pacemaker potential, resulting in an increase heart rate. D. Parasympathetic nervous system, excessive amounts of the hormone acetylcholine is released to the AV node by the parasympathetic nervous system, resulting in an increase of her heart rate. E. Parasympathetic nervous system, Polly is to relaxed in her new job, which releases the hormones epinephrine/ norepinephrine to the bundle of HIS where the pacemaker potential decreases, there is less K+ causing an increase in her heart rate.

c

393. Gilgamesh is a 20 year old male college student who has just been admitted into the trauma center at the OU Medical Center. He was involved in a car accident that left him unconscious. He finally wakes up at the hospital and begins to panic to the point of tears. The nurses measure his pulse to find that his heartrate has skyrocketed and his breathing has also became heavy and irregular. The doctors order them to sedate him. His heartrate declines after the injection of the drug and his breathing also returns back to normal. What kind of response was the patient having and how did the drugs work on his body to decrease his heartrate? How did the type of response affect his breathing rate? A. The patient was having a sympathetic response which triggered the release of epinephrine and norepinephrine which caused the elevated heart rate. The drug increased the affinity for the neurotransmitters to bind to the reuptake molecule, decreasing heartrate. The sympathetic response would increase breathing rate. B. The patient was having a parasympathetic response which triggered the release of acetylcholine which caused the elevated heart rate. The drug prohibited the reuptake molecule from taking up acetylcholine therefore decreasing heart rate. The parasympathetic response would increase breathing rate C. The patient was having a sympathetic response which is caused by the release of epinephrine and norepinephrine which caused the elevated heart rate. The drugs decreased his heart rate by stopping the sympathetic response by blocking release of epinephrine and norepinephrine and triggering a parasympathetic response to decrease heart rate. The sympathetic response would increase breathing rate. D. The patient was having a parasympathetic response which is caused by the release of epinephrine and norepinephrine which caused the elevated heart rate. The drugs decreased his heart rate by stopping the parasympathetic response by blocking the release of epinephrine and norepinephrine and triggering a sympathetic response to decrease heart rate. The parasympathetic response would decrease breathing rate. E. The Patient was having a sympathetic response which is caused by the release of epinephrine and norepinephrine which caused the elevated heart rate. The drugs decreased his heart rate by blocking the reuptake molecule from getting rid of the epinephrine and norepinephrine therefore decreasing heart rate. The sympathetic response would decrease breathing rate.

c

414. Johnny B. Goode has a history of vascular disease in his family. In which vessels is he most likely to have a problem: his arteries or his veins? a. Veins, because the vasa vasorum is restricted to the outer layers of veins, farther from the lumen, making it more difficult to nourish the cells of veins and remove waste products. b. Arteries, because the lower pressure (compared to veins) makes build up of wastes more likely. c. Arteries, because the vasa vasorum is restricted to the outer layers of arteries, farther from the lumen, making it more difficult to nourish the cells of arteries and remove waste products. d. Veins, because the higher pressure leads to venous tearing, which then nurtures infection. e. Veins, because the presence of valves in veins presents greater opportunity for infection and other medical problems.

c

415. The teaching assistant in your physiology lab just reached into an old dusty cabinet to fetch a microscope. As he grabs the microscope he lets out a sharp cry and says "something bite me!". Whatever was on the microscope has disappeared but your TA starts acting strange. He starts complaining of muscle cramps and is having a hard time breathing. What key ingredient are his muscles running out of to make ATP and what has happened to his sarcoplasmic concentration of calcium? A. His muscles are running out of glucose and his sarcoplasmic calcium concentration is nearing zero. B. His muscles are running out of glycogen and his sarcoplasmic calcium concentration is elevated. C. His muscles are running out of oxygen and his sarcoplasmic calcium concentration is elevated. D. His muscles are running out of glucose and his sarcoplasmic calcium concentration is elevated. E. His muscle are running out of oxygen and his sarcoplasmic calcium concentration is nearing zero.

c

418. Rachel woke up this morning feeling achy, nauseous, and dehydrated. She went clubbing last night with her friends and claims to have no recollection of the majority of the night. She had not had a single drink in a few months. They claimed that when she got home she ate a two-week-old piece of pizza, which must not have sat well with her stomach. As soon as she got out of bed she threw her pizza up. Why did Rachel black out last night? A. Rachel's drinks blocked the glutamate receptors in her frontal lobe which caused her memory to quit processing. B. The alcohol blocked the acetylcholine receptors in her frontal lobe which caused her to throw up. C. The alcohol in Rachel's drinks blocked the glutamate receptors within her hippocampus which altered the sympathetic division of her ANS which caused her to black out. D. The alcohol in Rachel's drinks blocked the glutamate receptors within her frontal lobe which altered the sympathetic division of her ANS which caused her to black out. E. The alcohol blocked the glutamate receptors in her frontal lobe which caused her to throw up.

c

419. Recently out of medical school, you are working as an intern at a local hospital. To prepare for your first patient of the day, Austin, you begin reading his chart. Austin has complaints of worsening peripheral vision and he has trouble fully exhaling the air from his lungs. What are possible causes and diagnoses of Austin's symptoms? A. Austin is suffering from Emmetropia and a common restrictive lung disease, chronic bronchitis. The emmetropia occurs when proteins die in the eye, clouding the cornea and resulting in poor peripheral vision while the chronic bronchitis restricts the lungs from expanding fully. B. Austin is suffering from glaucoma and a common restrictive lung disease, chronic bronchitis. The glaucoma occurs with a worsening flexibility of the lens while the while the chronic bronchitis restricts the lungs from expanding fully. C. Austin is suffering from glaucoma and a common obstructive lung disease, chronic bronchitis. The glaucoma occurs when there is a buildup in the aqueous humor of the eye, resulting in damage of the optic nerve and overall peripheral vision. The chronic bronchitis is restricting the airways of the lungs, resulting in difficulty exhaling fully. D. Austin is suffering from Emmetropia and a common obstructive lung disease, chronic bronchitis. The emmetropia occurs when there is a buildup in the aqueous humor of the eye, resulting in damage of the optic nerve and overall peripheral vision while the chronic bronchitis restricts the airways of the lungs, resulting in difficulty exhaling fully. E. Austin is suffering from glaucoma and a common obstructive lung disease, emphysema. The glaucoma occurs with a worsening flexibility of the lens while the emphysema restricts the lungs from expanding fully.

c

397. Suppose the nail in the head of the famous sea star to the right went through his brain and struck the front portion of the pituitary gland, but the back portion was fine. Which of the following is is correct? (Pretend the starfish has similar nervous and endocrine systems as humans do.) A. A primary endocrine disorder will occur where the protein hormone Growth Hormone (GH) will not be secreted from the anterior pituitary. Consequently Patrick's skeletal muscle will not make IGF-1 and proteogenesis in the skeletal muscle will not be promoted, so actin and myosin will be formed. B. A primary endocrine disorder will occur where the steroid hormone Growth Hormone (GH) will not be secreted from the anterior pituitary. Consequently Patrick's liver will not make IGF-1 and proteogenesis in the skeletal muscle will not be promoted, so actin and myosin will not be formed. C. A secondary endocrine disorder will occur where the protein hormone Growth Hormone (GH) will not be secreted from the anterior pituitary. Consequently Patrick's liver will make IGF-1 and proteogenesis in the skeletal muscle will be promoted, so actin and myosin will be formed. D. A secondary endocrine disorder will occur where the protein hormone Growth Hormone (GH) will not be secreted from the anterior pituitary. Consequently Patrick's liver will not make IGF-1 and proteogenesis in the skeletal muscle will not be promoted, so actin and myosin will not be formed. E. A secondary endocrine disorder will occur where the steroid hormone Growth Hormone (GH) will not be secreted from the anterior pituitary. Consequently Patrick's skeletal muscle will not make IGF-1 and proteogenesis in the skeletal muscle will not be promoted, so actin and myosin will not be formed.

d

403. Buddy the elf made himself breakfast consisting of candy, candy corn, candy cane, and syrup. He savored every sweet bite before it proceeded down into his digestive system. Which of the following is true based on the information given? (answer not provided) A. Excess acetyl CoA generated by lipolysis can be converted into fatty acids, triglycerides, cholesterol, steroids, and bile salts B. Lipids will be created from the acetyl CoA and take place in the mitochondria of adipocytes and hepatocytes C. The sweet flavor is a result from food molecules binding to a G protein-coupled receptor. D. Taste buds sense sweet tastes through ion channels triggered by ions in certain foods E. Buddy got diabetes from excess sugar intake due to his high levels of inulin not being able to keep up with the high glucose levels

e

404. Today is Martha's birthday. One of Martha's friends picked her up to take her shopping so that Martha's roommate could set up for the surprise birthday party they are throwing her. At the end of her shopping spree, they returned back to Martha's house and when she opened the front door and everyone yelled "SURPRISE!" she was so surprised that she turned around and ran out of the door. Which part of the nervous system was reacting to the surprise? What affect did this response have on her body? (WOULD HAVE TO REPHRASE BC EVERYTHING IS WRONG) A. Parasympathetic; bronchoconstriction, increased heart rate, increase in atrial depolarization, release of the neurotransmitter epinephrine from the postganglionic neuron that binds to adrenergic receptors B. Sympathetic; bronchodilation, decreased heart rate, increase in ventricular depolarization, release of the neurotransmitter epinephrine from the preganglionic neuron that binds to adrenergic receptors C. Parasympathetic; bronchoconstriction, increased heart rate, increase in ventricular repolarization, release of the neurotransmitter norepinephrine from the postganglionic neuron that binds to muscarinic cholinergic receptors D. Parasympathetic; bronchodilation, increased heart rate, decrease in atrial depolarization, release of the neurotransmitter acetyl choline from the postganglionic neuron that binds to nicotinic cholinergic receptors E. Sympathetic; bronchoconstriction, increased heart rate, increase in atrial depolarization, release of the neurotransmitter epinephrine from the postganglionic neuron that binds to adrenergic receptors

e

412. Which of the following statements is true about the phototransduction cascade? A. All-trans-retinal changes confirmation to 11-cis-retinal causing the alpha unit to slide over. B. When light triggers the cascade phosphodiasterase lowers cGMP levels causing Na+ influx and depolarization. C. Rods, which detect color vision, contain the protein retinal & retinal activates the shape change from 11-cis-retinal to all-trans-retinal. D. Phototransduction occurs in the outermost layer of the cornea, which helps regulate the amount of light entering the eye. E. When light triggers the cascade phosphodiasterase lowers cGMP levels causing hyperpolarization.

e

254. Which of the following could NOT be the result of excess cortisol levels? a. High blood glucose levels and accumulation of fat on face and neck b. "Moon face" as the result of Cushing's syndrome c. Decreases insulin production due to elevated blood sugar levels d. Over-activation of the sympathetic nervous system and decreased parasympathetic responses e. Vasodilation of blood vessels causing a decreased blood pressure

e

258. Which of the following is TRUE regarding sympathetic innervation of the Pacemaker cells of the Heart? a. Protein kinase A will phosphorylate the funny Na channels and T-type calcium channels. As a result, the openings of the channels are augmented causing a faster slow depolarization and increasing the heart rate. b. ACh will bind to muscarinic cholinergic receptors on the plasma membrane of the pacemaker cells activating the G- protein. The alpha sub unit of G protein will slide over and bump into the amplifier enzyme adenylate cyclase, which activates the second messenger system cAMP. c. The release of epinephrine when sympathetic nervous system is dominating will result in an increase in the number of pacemaker action potentials leading to an increase in heart rate. Epinephrine release will also result in bronchoconstriction. d. Epinephrine will bind to nicotinic cholinergic receptors causing the augmentation of funny sodium channels and L-type calcium channels resulting in an increased amount of pacemaker action potential. e. ACh binds to adrenergic receptors causing a decrease in blood resistance resulting in an increase in the frequency of pacemaker action potentials. This causes an increase in heart rate.

a

277. During a cardiac action potential, which of the following phases occurs when the voltage-gate calcium channels are open? a. Plateau b. Repolarization c. Rapid depolarization d. Brief repolarization e. None of these

a

283. Bert and Ernie were at the gym when all of a sudden, Ernie started hyperventilating and ended up fainting. Bert rushed Ernie to the emergency room and after many tests, the doctor determined that Ernie's plasma was very acidic and there was very little bicarbonate in it. What could have been the cause of this acidity and lack of bicarbonate in Ernie's plasma and what could have caused him to start hyperventilating? a. Ernie's body was making too many ketones due to excess Acetyl CoA and this ultimately led to him having Ketoacidosis. This meant that Ernie's plasma consisted of too many hydrogen ions and very little bicarbonate. The increased acidity and lack of bicarbonate led to a buildup of CO2 and this accounted for his hyperventilation. b. Ernie's body was not making enough ketones due to a lack of Acetyl CoA and this ultimately led to him having Ketoacidosis. This meant that Ernie's plasma consisted of too many hydrogen ions and very little bicarbonate. The increased acidity and lack of bicarbonate led to a lack of CO2 in Ernie's body and this accounted for his hyperventilation. c. Ernie's body was making too many ketones due to a lack of Acetyl CoA and this ultimately led to him having Ketoacidosis. This meant that Ernie's plasma consisted of too many hydrogen ions and very little bicarbonate. The increased acidity and lack of bicarbonate led to a buildup of CO2 and this accounted for his hyperventilation. d. Ernie's body was making too many ketones due to excess Acetyl CoA and this ultimately led to him having Ketoacidosis. This meant that Ernie's plasma consisted of very few hydrogen ions and bicarbonate. The increased acidity and lack of bicarbonate led to a buildup of CO2 and this accounted for his hyperventilation. e. Ernie's body was not making enough ketones due to excess Acetyl CoA and this ultimately led to him having Ketoacidosis. This meant that Ernie's plasma consisted of very few hydrogen ions and bicarbonate. The increased acidity and lack of bicarbonate led to a lack of CO2 in Ernie's body and this accounted for his hyperventilation.

a

284. Bob the electrician was working on a rainy day and did not notice the live wire next to the puddle and electrocuted himself, stopping his heart. Thankfully Bob was working at Tom the doctors' house. Tom was able to give bob a shot of epinephrine to get his heart beating again. Bob awoke suddenly however his heart rate was below 60 beats per minutes and his ECG showed no P wave. What is happening to bob? a. When bob was electrocuted it damaged the SA nod and the atria were no longer able to depolarize eliminating the pwave. This lowered his heart rate due to the AV node taking over his heart rate. b. When bob was electrocuted it damaged the AV node and the ventricles were no longer able to function which caused the p wave to be abnormal c. When bob was electrocuted it damaged the SA node and the ventricles were no longer able to depolarize eliminating the p wave. This lowered his heart rate due to the AV node taking over his heart rate. d. When bob was electrocuted it damaged the AV node and the atria were no longer able to depolarize eliminating the p wave. This lowered his heart rate due to the AV node taking over his heart rate. e. Tom the doctor accidently gave bob a shot of non - epinephrine which caused bob to have a slowed heart rate and the p wave to be eliminated.

a

289. On Friday the 13th Jasper decided to go to a haunted house. While in the haunted house many clowns, people will masks and chainsaws, and monsters popped out and scared him. Which of the following is TRUE regarding how Jasper's body would react in this scenario? a. He would have a sympathetic response, his body would release epinephrine and it would bind to beta-one receptors in the AV and SA nodes, and the ventricular myocardium, thus increasing his heart rate and increasing his force of contraction. b. He would have a sympathetic response, his body would release epinephrine and it would bind to beta-one receptors in the AV and SA nodes, and the atrial myocardium, thus increasing his heart rate and decreasing his force of contraction. c. He would have a parasympathetic response, his body would release epinephrine and it would bind to beta-one receptors in the AV and SA nodes, and the ventricular myocardium, thus increasing his heart rate and increasing his force of contraction. d. He would have a parasympathetic response, his body would release acetylcholine and it would bind to muscarinic cholinergic receptors in the AV and SA nodes. This would decrease his heart rate and allow him to not become frightened by the clowns. e. He would have a sympathetic response, his body would release epinephrine and it would bind to beta-one, beta-two, and alpha-one receptors all in the heart (AV, SA nodes, and ventricular myocardium) this would increase his heart rate and force of contraction.

a

298. Which of the following is true about the skeletal muscle? a. Acetylcholine is released from the axon terminal of the motor neuron and binds to nicotinic cholinergic receptors on the effector organ. b. Epinephrine is released from the axon terminal of the motor neuron and binds to muscarinic cholinergic receptors on the effector organ. c. Skeletal muscle can be dually innervated by both the parasympathetic system and the sympathetic system. d. The neuromuscular junction is between the varicosities and the effector and can create and End Plate Potential on the motor end plate. e. Skeletal muscle lines the uterus and the bladder and always has some contraction.

a

300. It is was a slow day in the ER, when all of the sudden, an ambulance rushes in with a young man named Will that is incased in cement, and it is slowly beginning to harden. The doctors are very worried about what is happening to his body while it is stuck in this cement. As the cement hardens, it is putting more and more weight on him. The cement also contains a certain chemical that acts as an antagonist for the neurotransmitter acetylcholine (Ach), and the longer he is in the cement, the more it diffuses into his organs. What are some of the possible physiological responses his body might be experiencing? A. As the cement hardens, the pressure that is being exerted on Will's body continues to increase. Due to this pressure, the alveolar pressure in his lungs will increase, and his lungs will not be able to inflate properly. It will get more difficult for him to breath, lowering the amount of oxygen he can get to his tissues. This would ultimately lead to his death. B. There will be an excess of Ach binding to the muscarinic cholinergic receptors on the SA node in his heart, causing his heart rate to slow drastically. C. As the cement hardens, the pressure that is being exerted on Will's body continues to increase. Due to this pressure, the volume of his thoracic cavity will decrease, causing the intrapleural pressure in his lungs to increase. Because of this, his lungs will not be able to inflate properly, causing a restrictive condition on his lungs. It will become more difficult for him to breath, lowering the amount of oxygen he can get to his tissues. D. Because Ach is involved with the sympathetic nervous system, his heart rate will not be able to increase because the chemical in the cement is blocking his adrenergic receptors. E. The chemicals that are seeping into his heart will alter the contraction of his ventricular myocardium.

a

310. Each of the following is incorrect regarding parasympathetic innervation of the pacemaker cells of the heart EXCEPT: A. A longer slow depolarization and repolarization phases will occur resulting a decreased heart rate. B. The L-type calcium channel will be suppressed resulting in a longer slow depolarization. C. An increased heart rate is the result of the suppression of the T-type calcium channels and augmenting the voltage gated potassium channels. D. Epinephrine will bind to beta 1 adrenergic receptors. Protein Kinase A will phosphorylate the funny sodium and T-type calcium channels resulting in the augmentation of their openings. This will result in a faster slow depolarization and a decrease heart rate. E. When parasympathetic response dominates, ACh reduces the pacemaker cell action potential leading to a decrease in heart rate, vasodilation, and bronchodilation.

a

319. A patient comes into your office complaining of having difficulty breathing, a fever, and chest pain. After running some tests you find that they have a pulmonary embolism in their pulmonary artery, what effects would this have on their heart rate and blood? A. Heart rate would increase due to an increase in resistance and less oxygen uptake in the blood along with less oxygen being transported to the tissues since blood flow is being impeded. With less oxygen there would be less CO2 being produced resulting in less CO2 in the blood to bind to bicarbonate and an unbalanced pH in the blood. B. Heart rate would decrease due to an increase in resistance and less oxygen uptake in the blood along with less oxygen being transported to the tissues since blood flow is being impeded. With less oxygen there would be more CO2 being produced resulting in more CO2 in the blood to bind to bicarbonate and a balanced pH in the blood. C. Heart rate would increase due to a decrease in resistance and more oxygen uptake in the blood along with more oxygen being transported to tissues since blood flow is being impeded. With more oxygen there would be less CO2 being produced resulting in less CO2 in the blood to bind to H+ ions to balance the blood pH. D. Heart rate would decrease due to a decrease in resistance and less oxygen uptake in the blood along with less oxygen being transported to the tissues since blood flow is being impeded. With less oxygen there would be less CO2 being produced in the blood to bind to bicarbonate and an unbalanced pH. E. Heart rate would increase due to a decrease in resistance and less oxygen uptake in the blood along with more oxygen being transported to tissues since blood flow is being impeded. With more oxygen being transported to the tissues there is less HCO3 binding to H+ ions to produce CO2 in the blood.

a

322. Sally has been training to run a marathon for over a year now. She can run up to 21 miles without stopping and believes that she is ready for the marathon and can run those extra 5.2 miles with no problem. On the day of the race, Sally was able to run the full length of the marathon and ran those extra 5.2 miles with ease. She claims that eating a granola bar rich in carbohydrates (mainly disaccharides) gave her the extra boost to finish the race. What allowed Sally to be able to run a full marathon and what might have caused the "extra boost"? A. Sally has a slow oxidative muscle fiber type which means that her muscles are slow to fatigue and have a high amount of mitochondria to generate more ATP for energy as she is running. The "extra boost" is due to the carbohydrates found in the granola bar. They are broken down into limit dextrins and maltose and then further into glucose monomers, with the help of enzymes of course, which can be absorbed into the bloodstream and used as a quick energy source. B. Sally has a fast glycolytic muscle type which means that her muscles are slow to fatigue and have a high amount of mitochondria to generate ATP for energy as she runs. The "extra boost" is due to the long chained carbohydrates in the granola bar. They are broken down into fructose and sucrose which can be absorbed into the blood stream and used as a quick energy source to finish the race. C. Sally was able to run the full length of the marathon because she ate a lot of carbohydrates for breakfast which allowed her to have just enough energy to get her thru most of the race. The granola bar she ate had even more long chained carbohydrates that were broken down into fructose and maltose which were able to be absorbed into the bloodstream to be used as a quick energy source so she could finish the race. D. Sally has a fast oxidative muscle fiber type which means that her muscles are slow to fatigue and have a high amount of mitochondria to generate more glucose to allow her to run for a longer period of time. The "extra boost" is due to the carbohydrates found in the granola bar. They are directly absorbed into the bloodstream and used as a quick energy source. E. Sally has a slow glycolytic muscle fiber type which means that her muscles are slow to fatigue and have a high amount of mitochondria to generate more ATP for energy as she is running. The "extra boost" is due to the carbohydrates found in the granola bar. They are broken down into limit dextrins and maltose and then further into glucose monomers that can be absorbed into the bloodstream and used as a quick energy source.

a

344. The 7 dwarfs were cooking cookies in the kitchen for Snow White. Grumpy thought it would be funny to trip Dopey, so he did. When Dopey was tripped, he fell head first into the corner of a cabinet. When Doc came to help, Dopey complained of blurry vision. When Doc was evaluating Dopey, he found a huge gash on his head from falling into the cabinet. Which of the following could not be the reason as to why he is having those symptoms? A. Dopey's gash is on the top of his head, so he damaged his occipital lobe B. When Dopey fell, he damaged his optic nerve C. Dopey damaged his cornea, which caused his light rays to not converge onto one focal point D. Dopey damaged his primary visual cortex on the back of his brain when he cut his head E. Dopey damaged his lens when falling, making his lens convex, causing his focal length to be shorter than normal

a

359. What is difference between systole and diastole (the two stages into which the cardiac cycle is divided)? A. Systole is the stage when the contraction of ventricular muscle fibers occurs and the ventricles are emptied. Diastole is the stage of the cardiac cycle when the ventricular muscles fibers expand and the ventricles are filled with blood. B. Systole is the stage when the contraction of atrial muscle fibers occurs and the atria are emptied. Diastole is the stage of the cardiac cycle when the atrial muscles fibers expand and the atria are filled with blood. C. Systole is the stage of the cardiac cycle when the ventricular muscles fibers expand and the ventricles are filled with blood. Diastole is the stage when the contraction of ventricular muscle fibers occurs and the ventricles are emptied. D. Systole is the stage of the cardiac cycle when the atrial muscles fibers expand and the atria are filled with blood. Diastole is the stage when the contraction of atrial muscle fibers occurs and the atria are emptied. E. Systole is the sound of the AV valves opening and closing. Diastole is the sound of the SL valves opening and closing.

a

361.Sarah, a 30-year old woman, traveled to the mountains to enjoy her winter break with friends and family for a few weeks. Shortly after arriving she began to notice that she would get tired from the littlest things such as climbing the stairs. With this, she also began to notice her heart beating very fast and shortness of breath. Which of the following explains why Sarah is experiencing these symptoms? A. Due to high altitudes and low air pressure in the mountains, it is harder for adequate levels of oxygen to be absorbed by the lungs. Because of this, less oxygen is reaching the blood stream, which causes the heart to beat harder and faster to deliver oxygen to the tissues. Due to the lack of oxygen the body gets tired easier. B. Due to high altitudes the lungs inhale more oxygen and exhale less carbon dioxide causing increase of respiratory activity and increased heart rate. C. In high altitudes oxygen becomes less available for our bodies. As well, air pressure outside of the body is higher than in the lungs, which causes it to become harder to inhale oxygen and pump it through the body. Due to this the heat begins to pump harder to compensate and causes fatigue. D. Because Sarah is visiting family and friends she has been enjoying lots of sodas, which contain high levels of caffeine. When the body is exposed to caffeine it increases parasympathetic reactions in the body. This causes increase in heart rate and respiratory activity, which would explain the shortness of breath and increase heart rate. E. High altitudes and low air pressure causes the IRV and ERV to increase greatly to get as much air in as possible and TV is decreased greatly.

a

364. Betsy once set the world record for fastest marathon runner but unfortunately now suffers from obesity. One night as she was walking home from work, she unluckily encountered a serial killer who made Betsy his target. At first, Betsy is able to outrun the killer but eventually slows down and is caught. What processes occurs in Betsy's body as she runs for her life, and what eventually keeps her from getting away successfully? a. Betsy's sympathetic nervous system was stimulated releasing NE/E, causing radial smooth muscles of the eyes to contract allowing more light to enter the eye as well as a shorter repolarization phase of the pacemaker potential. Betsy is not able to get away because of her restrictive lung disease and a low vital capacity, which wouldn't allow for enough air to enter her lungs. b. Betsy's parasympathetic nervous system was stimulated releasing Ach, causing circular smooth muscles of the eye to contract allowing more light to enter the eye as well as a decrease in the slow depolarization phase of the pacemaker potential. Betsy is not able to get away because of her obstructive lung disease and a low vital capacity, which wouldn't allow for enough air to enter her lungs. c. Betsy's sympathetic nervous system was stimulated, releasing NE/E, causing circular smooth muscles of the eye to contract allowing more light to enter the eye as well as a shorter repolarization phase of the pacemaker potential. Betsy is not able to get away because of her restrictive lung disease, which makes air getting out of the lungs difficult allowing for a buildup of CO2. d. Betsy's sympathetic nervous system was stimulated, releasing NE/E, causing radial smooth muscles of the eye to contract allowing more light to enter the eye as well as shorter repolarization phase of the pacemaker potential. Betsy is not able to get away because of her obstructive lung disease and a low vital capacity, which makes air getting out of the lungs difficult allowing for a buildup of CO2. e. Betsy's parasympathetic nervous system was stimulated releasing Ach, causing radial smooth muscles of the eyes to contract allowing more light to enter the eye as well as a decrease in the slow depolarization phase of the pacemaker potential. Betsy is not able to get away because of her restrictive lung disease, which causes the lining of the airway in patients to swell, shutting off the airways of the lung due to inflammation.

a

368. Jordan decided to go skydiving for her birthday. Unfortunately, she released her parachute too late, and she came crashing down into a tree. When she crashed into the tree, a branch impaled through her right arm and caused a significant amount of blood loss. When she finally arrived to the hospital in a very weak state, the ER attending physician determined that her blood pH was acidic, and she had low central venous blood pressure. What caused these symptoms to occur, and what is a possible response Jordan's body will have to combat these symptoms? a. Her blood is acidic because of the build up of lactic acid in the bloodstream. Since her organs are receiving less blood flow, it is causing the cells to utilize anaerobic respiration in order to produce enough ATP. Since there is a substantial decrease in blood volume, it is causing the rate of return of blood from the systemic veins to the heart to decrease. This has caused central venous blood pressure to be very low. In response, baroreceptors will be stimulated to detect the loss of blood volume and decreased systemic arterial blood pressure. Once baroreceptors have detected this, they will send messages to the brain to activate the sympathetic nervous system. b. Her blood is acidic because of an underactive adrenal gland. Since there is a substantial decrease in blood volume, it is causing the rate of return of blood from the systemic arteries to the heart to decrease. This has caused central venous blood pressure to become low. In response, baroreceptors will be activated to detect the loss of blood volume and increased systemic arterial blood pressure. Once baroreceptors have detected this, they will send messages to the brain to activate the sympathetic nervous system. c. Her blood is acidic because of the build up of lactic acid in the bloodstream. Her organs are receiving less blood flow, causing the cells to utilize anaerobic respiration in order to produce enough ATP. Since there is a substantial decrease in blood volume, it is causing the rate of return of blood from the systemic veins to the heart to increase. This has caused central venous blood pressure to be very low. In response, baroreceptors will be stimulated to detect the loss of blood volume and increased systemic arterial blood pressure. Once baroreceptors have detected this they will send messages to the brain to activate the parasympathetic nervous system. d. Her blood is acidic because of the build up of lactic acid in the bloodstream. Her organs are receiving less blood flow, causing the cells to undergo aerobic respiration in order to produce enough ATP. Since there is a substantial decrease in blood volume, it is causing the rate of return of blood from the systemic veins to the heart to reduce. This has caused central venous blood pressure to be very low. In response, baroreceptors will be activated to detect the loss of blood volume and decreased systemic arterial blood pressure. Once baroreceptors have detected this they will send messages to the brain to activate the parasympathetic nervous system. e. Her blood is acidic because of the build up of lactic acid in the bloodstream. Her organs are receiving less blood flow, causing the cells to utilize aerobic respiration in order to produce enough ATP. Since there is a substantial decrease in blood volume, it is causing the rate of return of blood from the systemic veins to the heart to increase. This has caused central venous blood pressure to be very low. In response, chemoreceptors will be activated to detect the loss of blood volume and decreased systemic arterial blood pressure. Once chemoreceptors have detected this they will send messages to the brain to activate the sympathetic nervous system.

a

372. Sally visited her physician complaining of brief 15-20 minute spells of heart palpitations, sweating, and nervousness. Her doctor decided to run a few tests during one of these spells and found that in addition to a rapid heart rate, Sally also presented with high blood pressure and increased blood sugar levels. What might be causing Sally's issues and what could her physician prescribe to alleviate her symptoms? a. An adenoma in Sally's adrenal gland may be causing hypersecretion of the catecholamines norepinephrine and epinephrine from the chromaffin cells. This over production increases sympathetic activity. To treat her symptoms, Sally's doctor may prescribe a sympatholytic drug, such as a Beta-blocker, to prevent epinephrine and norepinephrine from binding to adrenergic Beta receptors. b. An adenoma in Sally's hypothalamus may be causing hypersecretion of the hormones oxytocin and ADH from the pituitary gland. This over production increases sympathetic activity. To treat her symptoms, Sally's doctor may prescribe a sympatholytic drug, such as a Beta-blocker, to prevent epinephrine and norepinephrine from binding to adrenergic Beta receptors. c. A non-functioning adenoma in Sally's adrenal gland may be causing hyposecretion of the catecholamines norepinephrine and epinephrine from the chromaffin cells. This under production decreases sympathetic activity. To treat her symptoms, Sally's doctor may prescribe a sympathomimetic drug, such as an amphetamine, to bind to adrenergic Beta receptors and mimic the effects of epinephrine and norepinephrine. d. A non-functioning adenoma in Sally's hypothalamus may be causing hyposecretion of the hormones oxytocin and ADH from the pituitary gland. This under production decreases sympathetic activity. To treat her symptoms, Sally's doctor may prescribe a sympathomimetic drug, such as an amphetamine, to bind to adrenergic Beta receptors and mimic the effects of epinephrine and norepinephrine. e. An adenoma in Sally's adrenal gland may be causing hyposecretion of the catecholamines norepinephrine and epinephrine from the chromaffin cells. This under production increases sympathetic activity. To treat her symptoms, Sally's doctor may prescribe a sympathomimetic drug, such as an amphetamine, to bind to adrenergic Beta receptors and mimic the effects of epinephrine and norepinephrine.

a

373. Case Study: Susan comes into the hospital with high blood pressure. She also complains of having trouble with hearing, especially detecting which side she hears someone talk from. Upon further observation, her balance seems to be off also and her response to stimuli seems to be delayed. a. Susan has a problem in the medulla of her brain, causing neural signals in the ascending tracts to not function, and therefore autonomic responses to not function. This will cause the veins to not properly dilate during a sympathetic response and will therefore will decrease central venous return to the SVC or IVC and decrease central venous pressure. b. Susan has a problem with the medulla of her brain, causing neural signals in the ascending tracts to not function, and therefore autonomic responses to not function. This will cause the veins to not properly dilate during a sympathetic response and will therefore will increase central venous return to the SVC or IVC and decrease central venous pressure. c. Susan has a problem with the medulla of her brain, causing neural signals in the ascending tracts to not function and therefore autonomic responses to not function. This will cause auditory information coming from the vagus nerve to not synapse with neurons on the medulla and therefore sound localization will not occur. d. Susan has a problem with the medulla of her brain causing neural signals in the ascending tracts to not function and therefore atomic responses to not function. In addition, her proprioception will be thrown off and collateral projects from the medulla will not project to the cerebrum. e. Susan is experiencing metabolic acidosis from excess acidity. This resulted in decrease of reflexes and increase of muscle twitching. She will likely have headaches to follow. Her skin is flushed and her vessels are dilated.

a

287. Which of the following statements is false about Excitation-Contraction Coupling in Cardiac Contractile cells? a. Calcium binds to troponin and causes tropomyosin to shift exposing the myosin binding sites, which the myosin heads in their high energy state can then bind to, beginning cross bridge cycling. b. The pacemaker potential travels from the intracellular fluid of an adjacent autorythmic cell through a desmosome into the CCC, triggering calcium influx and eventually a cardiac potential. c. CCC's have a relative refectory period, however, it is so brief that it is considered non-existent. d. An action potential travels along the T-tubule of the sarcolemma to stimulate ryanodine to open and triggering calcium induced calcium release. e. Voltage gated sodium channels open, then voltage gated L type channels open, then slow calcium channels close, and finally the voltage gated potassium channels open in this order during the phases of a cardiac potential.

b

266. Which of the following statements is true about Calcium? a. Calcium binds to a nicotinic muscarinic receptors to create a cell response. b. High levels the normal of Ca in the SR can allow for more Ca to be released, causing more Ca to bind to troponin resulting in more muscle contractions. c. Ca in a resting cell has a low concentration inside the cell and high concentration outside the cell. d. NE/E bind to alpha one receptors to activate IP3 second messenger system to allow for arties to vasoconstriction due to low levels of Ca, which causes high TPR and high MAP. e. Ca binds to Calmodulin in order to stop smooth muscle from contracting.

b

292. Charlie Sheen wakes up in the middle of a dark room with slight amnesia. Sitting in front of him are five pills, each labeled with a notecard describing what is in each pill. Mr. Sheen notices that his heart is beating unusually fast (even more frantically than when he is on one of his cocaine binges), that he is profusely sweating, and he is starting to become woozy. Which of the pills should Mr. Sheen take to try to decrease his heart rate and why should he choose it? A. Atropine - Charlie should choose this pill because atropine is a widely known muscarinic antagonist that will stimulate the parasympathetic innervated locations of the myocardium and nodes which will result in a decreased heart rate. B. Pilocarpine - Charlie should choose this pill because pilocarpine is a widely known parasympathomimetic which mimics acetylcholine and induces a response from muscarinic cholinergic receptors in the parasympathetic innervated portions of the myocardium and nodes leading to an overall decreased heart rate. C. Epinephrine - Charlie should choose this pill because epinephrine is one of the most widely known parasympathetic neurotransmitters which will induce responses from adrenergic receptors the parasympathetic innervated portions of the myocardium and nodes of the heart leading to an overall decrease in heart rate. D. Albuterol - Charlie should choose this pill because albuterol is a widely known parasympathomimetic which mimics acetylcholine's' action on the nicotinic cholinergic receptors located in the parasympathetic innervated portions of the myocardium and nodes of the heart, leading to a decrease in Charlies' overall heart rate. E. Isoproterenol - Charlie should choose this pill because isoproterenol is a widely known sympathomimetic drug which induces responses from the beta-1 adrenergic receptors located in the sympathetic innervated portions of the myocardium and nodes of the heart, leading to an overall decrease in heart rate.

b

297. Joe has just started working out and notices how sore his legs are as he is walking to his physiology class. In physiology he is learning about the heart and how it works. He learns that the heart is a muscle too and that is why the heart must contract to be able to pump blood throughout the body. This new knowledge has Joe thinking, if the heart is a muscle and it is constantly contracting to pump blood then why does it never fatigue and feel sore like his legs did after his workout? Joe also wonders while he is working out how his breathing rate is maintained and what detects his low oxygen levels? A. The type of muscle in Joe's legs is skeletal and the type of muscle in Joe's heart is smooth muscle. Smooth muscle is known to constantly be a little contracted at all times and this helps it not fatigue. Skeletal muscle fatigues easier because if it does not have a constant steady supply of oxygen it cannot keep doing aerobic respiration to make ATP to fuel the muscle during a workout and the muscle fatigues. The cerebellum maintains Joe's breathing rate and peripheral chemoreceptors detect low oxygen levels. B. The type of muscle in Joe's legs is skeletal and the type of muscle in his heart is cardiac muscle. Cardiac muscle is able to constantly contract without fatiguing because of the large amounts of mitochondria and myglobin it has to make ATP to fuel the muscle. Skeletal muscle fatigues easier because it does not have as many mitochondria and myoglobin as cardiac and if there is not enough oxygen to perform aerobic respiration not enough ATP will be made to fuel the muscle and it eventually fatigues. The medulla and the pons control his breathing rate and peripheral chemoreceptors detect low oxygen levels. C. The type of muscle in Joe's legs is skeletal and the type of muscle in his heart is cardiac. Cardiac muscle is able to constantly contract without fatiguing because it constantly has oxygen being supplied to it. Skeletal muscle does not have as much oxygen being supplied to it and fatigues faster because there are no more available troponin binding sites. The medulla and the pons control Joe's breathing rate and central chemoreceptors detect low oxygen levels. D. The type of muscle in Joe's legs is smooth and the type of muscle in Joe's heart is cardiac. Cardiac muscle is able to constantly contract without fatiguing because it has large amounts of mitochondria to make ATP to fuel the muscle. Smooth muscle fatigues faster because it does not have many mitochondria. The pons controls his breathing rate and the baroreceptors detect low oxygen levels. E. The type of muscle in Joe's legs is skeletal muscle and the type of muscle in his heart is cardiac. Cardiac muscle is able to constantly contract without fatiguing because is has a small resting period in between contractions and has a large amount of mitochondria to make ATP to fuel the muscle. Skeletal muscle does not posses this and fatigues faster. Joe's breathing rate is controlled by the medulla and pons and the baroreceptors detect low oxygen levels.

b

299. Which of the following is FALSE pertaining to epinephrine? A. Receptors for epinephrine in the heart are located at the SA node, the AV node, and the ventricular myocardium. B. Epinephrine is lipid soluble. Therefore, it binds to beta one adrenergic receptors inside the target cell. C. Epinephrine is released by the medulla layer of the adrenal glands during a sympathetic response. D. Epinephrine is released by the postganglionic neuron where it will then bind to the effector organ. E. When released, epinephrine will affect the heart by increasing the frequency of cardiac action potentials and therefore increasing heart rate by activating the cAMP second messenger system to cause a shorter slow depolarization and repolarization in the pacemaker action potential.

b

304. Suzy was outside playing soccer with her friends. After running around and kicking the ball she noticed shortness in her breath. As she kept playing it became even more difficult for her to breath. She eventually started coughing and wheezing and she felt a lot of pressure in her chest. Suzy started feeling panicked so she quickly called her mom who is conveniently a doctor! What did Suzy's mom do to solve the problem and save her daughter? A. Suzy is going into cardiac arrest so her mom told her to inject herself with epinephrine. The epinephrine will induce a sympathetic response which will allow for increased pressure of the coronary artery and increased blood flow from the heart. The blood flow will decrease her heart rate and allow her to breath easier. B. Suzy is having an asthma attack so her mom told her to use her inhaler which contains epinephrine. The epinephrine will cause bronchodilation. This will open her airways allowing her to breath better. C. Suzy is just out of shape so her mom told her to sit down and take a break. Suzy was releasing a lot of epinephrine from her hypothalamus which was causing her to have an increased heart rate. After Suzy caught her breath she was able to breath normal again. D. Suzy is having an asthma attack so her mom told her to use her inhaler which contains epinephrine. The epinephrine will bind to beta one adrenergic receptors which will cause a sympathetic response and her airways will open so she can breath better. E. Suzy is going into cardiac arrest so her mom told her to inject herself with epinephrine. The epinephrine binds to alpha one receptors and triggers the second messenger cAMP which will cause calcium to be released and bind to calmodulin. This will open the airways and allow Suzy to breath better.

b

305. Frieda Livery, a 27 year old female, pregnant with her first child is 4 days past her due date, in August. Her doctor decides to administer an exogenous drug named Pitocin that functions by mimicking the effects of Oxytocin, a neurohormone produced by the hypothalamus. Pitocin will _______: A. bind to receptors on the single-unit smooth muscle of the uterus, causing contractions of discrete functional units, which will result in the birth of the child, at which point negative feedback will turn off the system. B. bind to receptors on the single-unit smooth muscle of the uterus, causing a functional syncytium of contractions, which will continue using a positive feedback loop, resulting in the birth of the child. C. bind to receptors in the posterior pituitary causing a neurogenic response in the single-unit smooth muscle of the uterus, causing a functional syncytium of contractions, which will continue using a positive feedback loop, resulting in the birth of the child. D. bind to receptors on the multi-unit smooth muscle of the uterus, causing contractions of discrete functional units, which will continue using a positive feedback loop, resulting in the birth of the child. E. bind to receptors on the single-unit smooth muscle of the uterus, causing a functional syncytium of contractions, which will result in the birth of the child, at which point negative feedback will turn off the system.

b

314. Little Cindy Lou Who was helping her dad in the mailroom, and fell off the ladder onto a pile of presents. She landed just right so that the corner of one of the presents in the pile fractured two of her ribs. Which of the following statements about her condition would be considered FALSE? A. After taking a blood sample in Cindy Lou's brachial artery, they could see that her oxygen levels would be low and her carbon dioxide levels would be high, meaning that her capillaries are not functioning properly in her lungs. B. Cindy Lou would have hypertension, because she is in such shock of what just happened. C. Measuring Cindy's heart rate they determined that she had tachycardia. D. Loss of breath due to the atmospheric air rushing into the hole on the side of her body. She is having a hard time with expiration keeping up due to the large amounts of inhalation and air rushing in. E. Poor Little Cindy Lou could potentially go into shock and pass out because she is not getting enough air into the collapsed lung, and instead as an increase in air around her lungs.

b

328. Steve, The Safari Man, takes weekly safari trips. One week, Steve and five of his lucky fans set out into wild in his very own Land Rover. As they were setting out on the adventure, they spotted a lion sleeping near the famous "Tree of Life." Since it was sleeping, Steve thought that he could give his fans a closer look at the animal if they were to tranquilize the lion for a few hours. So that's just what he did. CLICK! The tranquilizing gun released the serum that would keep the lion asleep. In this moment Steve realized that the tranquilizer was expired. Suddenly, the lion woke up to the pinch of the needle, and was looking for his attackers. The crew prepared for their next move. In this moment what occurred to Steve and his five not so lucky fans? A. The parasympathetic nervous system was activated; Ach was released from the pre-ganglionic neuron onto the muscarinic receptors located on the post-ganglionic neuron; the post-ganglionic neuron released epinephrine onto adrenergic receptors causing a "fight or flight" response; the parasympathetic response caused the bronchioles to dilated which allowed more carbon dioxide to reach the tissues to assist in running away from the lion B. The sympathetic nervous system was activated; Ach was released from the pre-ganglionic neuron onto nicotinic receptors located on the post-ganglionic neuron; the post-ganglionic neuron released epinephrine onto adrenergic receptors causing a "fight or flight" response; the sympathetic response caused the bronchioles to dilate which allowed more oxygen to reach the tissues to assist in running away from the lion C. The sympathetic nervous system was activated; Ach was released from the pre-ganglionic neuron onto nicotinic receptors located on the post-ganglionic neuron; the post-ganglionic neuron released epinephrine onto cholinergic receptors causing a "fight or flight" response; the sympathetic response causes the Beta 1 receptors on the cardiac muscle to increase the heart rate which pumped more blood throughout the body to assist in running away D. The sympathetic nervous system was activated; Epinephrine was released from the pre-ganglionic neuron onto nicotinic receptors located on the post-ganglionic neuron; the post-ganglionic neuron released Ach onto adrenergic receptors causing a "fight or flight" response; the sympathetic response caused the pupils to constrict to assist in running away from the lion E. The sympathetic nervous system was activated; Norepinephrine was released from the pre-ganglionic neuron onto nicotinic receptors located on the post-ganglionic neuron; the post-ganglionic neuron released epinephrine onto adrenergic receptors causing a "fight or flight" response; the parasympathetic response caused the bronchioles to constrict; when the bronchioles constricted it allowed for more oxygen to reach the tissues to assist in running away from the lion

b

329. Uh oh! Dr. Ketchum was holding a tutoring session for the next physiology exam when a mysterious spider bit her! She goes to the doctor to get checked out and they notice that this bite has caused her potassium channels to stay open. Which of the following would happen within Dr. Ketchum's body? a. Action potentials will fire more rapidly due to increased potassium efflux caused by and increase of potassium in the extracellular fluid. b. She will be more sensitive to loud noises because potassium will be constantly flowing into her stereocilia, causing more neurotransmitter to be released and more action potentials will be fired to her brain. c. Carlisle's pacemaker potentials will fire much faster due to hyper polarization. d. Carlisle's eyes will be more sensitive to light. e. Carlisle's stereocilia are going to bend so far that they break.

b

340. You drink a lot of coffee and chase it with red bull which inhibits phosphodiesterase. Does this increase or decrease levels of cAMP? Four pumpkin spice lattes and twenty one dollars later you're leaving the Physiology final to head to the bathroom. What correlation does an excess of liquid have with the kidneys? A. Levels of cAMP would increase because phosphodiesterase breaks down cAMP and since caffeine inhibits phosphodiesterase cAMP would be higher than normal. Caffeine inhibits ADH which is Antidiuretic hormone so it causes less frequent urination B. Levels of cAMP would increase because phosphodiesterase breaks down cAMP and since caffeine inhibits phosphodiesterase cAMP would be higher than normal. Caffeine inhibits ADH which decreases the the water absorption in the kidneys and causes frequent urination. C. Levels of cAMP would be lower than normal because phosphodiesterase inhibits the production of cAMP. Caffeine inhibits ADH which is Antidiuretic hormone so it causes less frequent urination. D. Levels of cAMP would be lower than normal because phosphodiesterase inhibits the production of cAMP. Caffeine inhibits ADH which decreases the the water absorption in the kidneys and causes frequent urination E. There would be no increase or decrease in cAMP because phosphodiesterase does not have an effect on cAMP. Caffeine and kidneys do not correlate.

b

355. Which of the following statements are false about the cardiac system? A. The pulmonary arteries are responsible for carrying deoxygenated blood into the lungs, while the pulmonary veins bring oxygenated blood back into the heart. B. Blood will flow from the superior vena cava, inferior vena cava, or coronary sinus into the right atrium through the ventricular semilunar valve and into the right ventricle. C. During a sympathetic response norepinephrine/epinephrine will bind to B adrenergic receptors on the SA and AV nodes. D. In an ECG, the QRS complex is the depolarization of the ventricles and the repolarization of the atria. E. The "Dupp" sound occurs during the closing of the semilunar valves.

b

370. A ________ synapse releases neurotransmitters are released into the ________ through a process called ________. a. Chemical; gap junction; endocytosis. b. Chemical; synaptic cleft; exocytosis. c. Electrical; gap junction; exocytosis. d. Chemical; synaptic cleft; reception. e. Electrical; synaptic cleft; endocytosis.

b

375. Which of the following is false about Epinephrine? a. It is secreted by chromaffin cells located in the adrenal medulla. b. It will bind to adrenergic receptors and create a fast response. c. It is a type of catecholamine and therefore derived from tyrosine. d. It will bind to a receptor located on the radial muscle of the iris creating a sympathetic response and allowing the pupil to dilate. e. It affects the heart and will use a second messenger system activating cAMP.

b

376. Dr. Ketchum has been preparing to show her horse in the national championship show yet again. Everyone in the United States is looking forward to seeing THE Dr. Ketchum take home the gold for the third year in a row. It's one week before the big day and Dr. Ketchum is starting to worry because her horse is nowhere to be found! She has come to the conclusion that her star horse has been stolen out of her trailer. Dr. Ketchum is so angry that her whole body is shaking and she feels as though her heart is going to beat out of her chest as she is trying to figure out what to do with nationals only one week away. Which of the following is NOT a correct statement? a. Dr. Ketchum was experiencing a sympathetic response after realizing that her horse has been stolen. This included the innervation of her cardiac muscles, smooth muscles, and glands in order to prepare her body for flight-or-flight response. b. Dr. Ketchum's body was performing glucogenolysis, gluconeogenesis, lipolysis, and ketogenesis once epinephrine was synthesized and secreted throughout her body. c. As Dr. Ketchum is trying to figure out a plan to find her horse, her heart rate is elevated and the force of contraction of her heart has increased. This is due to epinephrine binding to adrenergic receptors on the SA node, AV node, and the ventricular myocardium. d. Epinephrine was released from the chromaffin cells in the adrenal medulla which then bound to Alpha 1 receptors to activate the IP3 second messenger system. This allowed for vasoconstriction in order to decrease total peripheral resistance (TPR). e. Dr. Ketchum's body was able to recognize that her heart rate was above normal due to her baroreceptors sending an increased number of action potentials to the cardiovascular control center in the medulla.

b and d

257. You work in a clinic and have an interesting lady comes in to your office. She complains that she is very on "edge" and is startled easily lately. She feels like she can never calm down and wants help before her daughter's wedding. She states that since the father is dead she will be walking her daughter down the aisle and she would like to do it without having shaky hands. She has been experiencing these symptoms for a week. You take her heart rate and find that it is at 135 beats per minute. Is this heart rate normal? If not, what kind of response is this women stuck in? What hormone is being either hyper or hypo excreted? What type of receptor does this hormone use? Is this receptor slow or fast? How does this hormone affect her heart rate, if it is not normal? (Increase or decrease). Which organ secretes this hormone and where is it located? What type of medicine can you prescribe to help this woman's case? Which of the following answers are most correct: a. This heart rate is not normal. She is in a sympathetic response. ACH is being hyper excreted, and uses an adrenergic receptor. It binds to the adrenergic receptor which is slow ligand gated. This hormone will decrease her heart rate. The adrenal glands located around the lungs secrete this hormone. You can prescribe a sympathomimetic drug. b. This heart rate is normal and she is just nervous for her daughter's wedding. She is only overreacting. c. Her heart rate is not normal. She is experiencing a sympathetic response. Epinephrine is being hyper excreted. This hormone uses an adrenergic receptor and is slow ligand gated. This hormone will increase her heart rate. The adrenal glands secrete this hormone and are located on top of the kidneys. You can prescribe a beta blocker to help with these symptoms. d. Her heart rate is not normal. She is experiencing a sympathetic response. Epinephrine is being hypo excreted. This hormone uses an adrenergic receptor and is fast ligand gated. This hormone will increase her heart rate. The adrenal glands secrete this hormone and are located on top of the kidneys. You can prescribe an anticholinergic medicine to help with these symptoms. e. Her heart rate is not normal. She is experiencing a sympathetic response. Epinephrine is being hyper excreted. This hormone uses an adrenergic receptor and is slow ligand gated. This hormone will decrease her heart rate. The adrenal glands secrete this hormone and are located on top of the stomach. You can prescribe a beta blocker to help with these symptoms.

c

262. When running away from a mugger,_________ stimulation causes the hormone________ to bind to ________ receptors in the heart and causes hydrolysis of ATP on the myosin head to occur at a _______ rate, resulting in the rate of cross bridge cycling to _______. a. Parasympathetic, Acetylcholine, Muscarinic Cholinergic, slower, decrease b. Sympathetic, Acetylcholine, Nicotinic Cholinergic, faster, remain unchanged c. Sympathetic, Epinephrine, beta-one adrenergic, faster, increase d. Sympathetic, Epinephrine, alpha-one adrenergic, slower, remain unchanged e. Sympathetic, Norepinephrine, beta-three adrenergic, slower, decrease

c

263. Which of the following is not true? (NO ANSWER GIVEN) a. Sympathetic preganglionic cell bodies are found in thoracic spinal cord lateral horns. b. Sympathetic nerves innervate the smooth muscle in arteries and veins c. Parasympathetic nerves innervate arrector pili and blood vessels in the dermis d. Sympathetic ganglia are usually located near the spinal cord e. The sympathetic division of the ANS is known as the thoracolumbar division

c

279. Helen is working the night shift as a mall cop at the Sooner Mall when she hears a gunshot. Not being a very good mall cop, she gets scared and begins to run away. Which hormone induced Helen's response and what effect does this have on her respiratory and cardiovascular systems? a. Acetylcholine is released from the motor end plate, increasing the frequency of action potentials, which would increase Sally's heart rate, therefore increasing oxygen delivery to the systemic tissues. b. E/NE is released from the adrenal medulla, causing her bronchioles to dilate so oxygen delivery to the systemic tissues can increase, and causing her veins to constrict, which increases resistance, MAP, and blood flow. c. E/NE is released from the adrenal medulla, causing her bronchioles to dilate which will increase oxygen delivery to the systemic tissues, and causing her veins to dilate, which decreases resistance, and increases blood flow. d. E/NE is released from the adrenal cortex, causing dilation of the bronchioles to increase oxygen intake and causing constriction of the veins to increase blood flow. e. E/NE is released from the adrenal cortex, increasing Sally's heart rate which increases oxygen delivery to the tissues and causing expansion of her lungs to increase cellular respiration.

c

281. What effect on blood pressure does an increase in vasopressin have and how is blood pressure returned to normal? a. Increased vasopressin causes more water and sodium reabsorption from the kidneys into the blood, which increases the blood volume and in return, increases stroke volume and therefore, increases the mean arterial pressure. Chemoreceptors in the carotid arteries and aortic arch sense the increased pressure and relay to the medulla to signal precapillary sphincters to vasodilate, as well as the heart to slow down. b. Increased vasopressin causes more water and sodium excretion from the kidneys out of the body, which decreases the blood volume and in return, decreases stroke volume and therefore decreases the mean arterial pressure. Baroreceptors in the carotid arteries and aortic arch sense the decreased pressure and relay to the medulla to signal precapillary sphincters to vasoconstrict, as well as the heart to speed up. c. Increased vasopressin causes more water and sodium reabsorption from the kidneys into the blood, which increases the blood volume and in return, increases stroke volume and therefore increases the mean arterial pressure. Baroreceptors in the carotid arteries and aortic arch sense the increased pressure and relay to the medulla to signal precapillary sphincters to vasodilate, as well as the heart to slow down. d. Increased vasopressin causes less water and sodium reabsorption from the kidneys into the blood, which increases the blood volume and in return, increases stroke volume and therefore, increases the mean arterial pressure. Baroreceptors in the carotid arteries and aortic arch sense this increased pressure and relay to the medulla to signal precapillary sphincters to vasodilate, as well as the heart to slow down. e. Increased vasopressin causes less water and sodium reabsorption from the kidneys into the blood, which decreases the blood volume and in return, decreases stroke volume and therefore decreases the mean arterial pressure. Chemoreceptors in the carotid arteries and aortic arch sense the decreased pressure and relay to the medulla to signal precapillary sphincters to vasoconstrict, as well as the heart to speed up.

c

282. Which of the following statements is true regarding action potentials? a. In the heart, rapid depolarization of the cardiac contractile cells is caused by the opening of calcium channels. b. In a pacemaker cell, there is a stable resting potential so the opening of Na+ channels does not effect the opening of Ca2+ channels. c. Cardiac contractile cells have essentially no relative refractory period where in other cells there is an absolute and relative refractory period. d. In action potentials in skeletal muscle or neurons an action potential is generated when the membrane potential reaches -80mV. e. Action potentials occur in the same fashion across all cells of the body in regards to the threshold potential, length and duration of phases, and the channels involved.

c

285. You have a big physiology final coming up, which makes you very nervous and stressed. You have increased your coffee intake in an effort to stay up later, drinking multiple cups a day. How will drinking more coffee affect your cardiac output? a. Ingested caffeine will bind to beta-one adrenergic receptors, causing the cAMP second messenger system, causing more action potentials to be sent, more muscle contractions, and thus more pumping of blood. This will increase cardiac output. b. Epinephrine will bind to beta-one adrenergic receptors, causing the cAMP second messenger system to begin. Caffeine stimulates the action of protein kinase A, causing more protein phosphorylation, which will cause more action potentials, and more pumping of blood. This will increase cardiac output. c. Epinephrine will bind to beta-one adrenergic receptors, causing the cAMP second messenger system to begin. Caffeine inhibits the action of the cAMP phosphodiesterase, which prevents cAMP from breaking down. With more cAMP present, there will be more action potentials and more pumping of blood. This will increase cardiac output. d. Ingested caffeine will inhibit epinephrine from binding to beta-one adrenergic receptors. This will inhibit action potentials from occurring as frequently, causing low muscle contraction, and lowering cardiac output. e. Epinephrine will bind to alpha-one adrenergic receptors, causing the IP3 second messenger system to begin. Caffeine inhibits the action of calmodulin, creating less muscle contractions, so less blood will be pumped. This will lower cardiac output.

c

291. William, a 4-year-old boy was rushed into Norman Regional Hospital today with a variety of problems. Poor William was extremely fatigued, had a dry cough, shortness of breath, red bumps on his face, and an enlarged liver. He complains of not being able to get as much air in as he can and feels as if it is being stopped half way through the breath. Unfortunately that is not the last of his problems. Along with his newly diagnosed respiratory disorder, the doctors also found that his heart rate was increasing at a very large amount and his body was not reacting properly. From this information, what kind of respiratory disease does he have and a possible condition from that category of disease? Also, what response should his body have performed to fix the problem of his increasing heart rate? A. Obstructive-emphysema; sympathetic response-the postganglionic cell should have released Ach and bound to a muscarinic receptor. B. Obstructive-asthma; sympathetic response-the postganglionic cell should have released Epinephrine and bound to a adrenergic receptor C. Restrictive-sarcoidosis; parasympathetic response-the varicosities on the postganglionic cell should have released Ach and bound to a muscarinic cholinergic receptor on the effector D. Restrictive-chronic bronchitis; parasympathetic response- the varicosities on the postganglionic cell should have released Epinephrine and bound to a adrenergic cholinergic receptor on the effector E. Restrictive-obesity; sympathetic- the postganglionic cell should have released Norepinephrine and bound to a muscarinic receptor.

c

293. Jim walks into the emergency room with partial flaccid paralysis and a rapid heartbeat. He informs his doctor that he is in an experimental drug group and taking drug XZ219. After examination the doctor observes that Jim's skeletal muscles are not contracting properly, and that his heartbeat is elevated above the normal range. What effect could this drug be causing? A. Drug XZ219 is completely inhibiting Acetylcholine release which prevents activation of Muscarinic Cholinergic receptors in skeletal muscles and prevents a parasympathetic response of the heart by preventing activation of Muscarinic Cholinergic receptors in the heart. B. Drug XZ219 is partially inhibiting Epinephrine release which partially prevents activation of Nicotinic Cholinergic receptors in skeletal muscles and prevents a parasympathetic response of the heart by partially preventing activation of Muscarinic Cholinergic receptors in the heart. C. Drug XZ219 is partially inhibiting Acetylcholine release which partially prevents activation of Nicotinic Cholinergic receptors in skeletal muscles and prevents a parasympathetic response of the heart by partially preventing activation of Muscarinic Cholinergic receptors in the heart. D. Drug XZ219 is completely inhibiting Acetylcholine release which prevents activation of Nicotinic Cholinergic receptors in skeletal muscles and prevents a sympathetic response of the heart by preventing activation of Muscarinic Cholinergic receptors in the heart. E. Drug XZ219 is increasing Acetylcholine release which causes activation of Nicotinic Cholinergic receptors in skeletal muscles and causes a parasympathetic response of the heart by activation of Muscarinic Cholinergic receptors in the heart.

c

295. Which of the following is NOT an example of a function of the sympathetic division of the autonomic nervous system? A. Stimulates release of Acetylcholine at the axon terminals of preganglionic fibers which bind to nicotinic cholinergic receptors on the postganglionic fiber. B. Influences various organ systems of the body through connections emerging from thoracic and upper lumbar spinal cord. C. Project to the adrenal cortex which stimulates chromaffin cells to release epinephrine and norepinephrine D. Causes the digestive system to shut down most of its function in order to direct blood flow to the skeletal muscles E. Sympathetic nervous system responses cause heart rate to increase via sympathetic fibers that innervate the SA node, AV node, both atria, and both ventricles, which directly stimulate the heart by increasing the production of cAMP second messenger, thereby increasing the rate at which action potentials are fired in the heart.

c

296. Which of the following might occur if someone could no longer use cAMP efficiently? A. The funny channels and T-type calcium channels would be phosphorylated less often, resulting in a faster depolarization phase. The repolarization phase will also be faster because the potassium channels will close sooner. B. Less thyroid hormone would be brought into the blood, causing a decrease in body heat production and a decrease in body weight. C. Potassium leak channels would be phosphorylated less frequently during slow EPSPs, making the membrane potential of the postsynaptic neuron less positive and decreasing the chance of generating an action potential. D. There would be less vasodilation of arteriolar smooth muscle because the alpha-one adrenergic receptors in vascular smooth muscle could no longer activate the cAMP second messenger system. E. Muscarinic cholinergic receptors in the sympathetic nervous system would have less efficiency in generating a response on effector organs because the ion channels would open/close less.

c

301. It was the first day of Anatomy, second semester, when the class was required to take a "quiz" about everything they learned the previous semester in Physiology. Troy was on question 17 "Put the structures of the body, in terms of fundamental levels of organization in order by increasing in complexity.", when he started to have a pain in his chest. Troy immediately got up from his seat and went straight to the ER without finishing the quiz, due to his family's history of heart problems. At the hospital they quickly set him up to an EKG, and the reading proved to be abnormal. There was a long delay between the P-wave and the QRS complex. The doctors saw that and knew they needed to do something quick. For the first part of this question, answer question 17 for Troy, then determine what the delay between the P-wave and the QRS complex means. If Troy wasn't to have gone to the hospital, and the pain got worse, and his breathing continued to slow, what would happen? A. Subatomic particles, atoms, molecules, cells, tissues, organs, organisms, organ system, biosphere; 1st degree partial block indicating a delay in connection between the atria and ventricles; It would just go away. B. Subatomic particles, atoms, molecules, organelles, cells, tissues, organs, organisms, organ system, biosphere; 1stdegree partial block indicating a delay between the left ventricle and the aorta; if it increased fast enough he would have a complete heart block and die. C. Subatomic particles, atoms, molecules, organelles, cells, tissues, organs, organisms, organ system, biosphere; 1stdegree partial block indicating a delay in connection between the atria and ventricles; if it increased fast enough he would have a complete hear block and die. D. Atoms, cells, tissues, organs, organisms, organ system; 2nd degree partial block indicating a delay in connection between the atria and ventricles; the pain may stay but he can still do everything normally. E. Cells, tissues, organs, organisms, organ system, biosphere 2nd degree partial block indicating a delay in connection between the arteries and veins; if it increased fast enough he would have a complete heart block and die.

c

306. Which of the following is true regarding the human endocrine system? A. Gonadotropin-releasing hormone is synthesized and secreted from the hypothalamus, which triggers a release of luteinizing hormone from the anterior pituitary, and then a release of testosterone from the testes. This release of testosterone causes an increase in number of skeletal muscle fibers in males. B. Oxytocin is released from the anterior pituitary and targets the uterus and mammary glands in women, and is helpful in the bonding of a mother and her child. C. Chromaffin cells are secretory cells in the adrenal medulla. These cells are used in sympathetic innervation of the heart to release epinephrine and norepinephrine that bind to beta-one adrenergic receptors in the heart. This binding causes heart rate and ventricular contraction to increase. D. An excess of calcium in the blood causes parathyroid hormone (PTH) to be synthesized and secreted by the thyroid gland. PTH inhibits osteoclasts to avoid bone being broken down and causes calcium to be excreted through urine via the kidneys. E. A hyposecretion of thyroid hormone is primary disorder of the endocrine system that causes a decreased amount of the hormone. This causes an increase in basal metabolic rate and heart rate.

c

307. Mallory, a cheerleading coach, is experiencing weakness in her limbs that comes and goes. Over time, she has been complaining that her weakness in her limbs has become increasingly worse. Knowing this, it has been difficult for Mallory to spot her athletes on acrobatic tumbling skills due to her weakness in her arms. While she has been at work, she has also been noticing a shortness of breathe. Mallory has been diagnosed with a disease that affects her sensory and motor neurons from an excess amount of acetylcholine in her body. Choose the correct statement. A. Mallory's disease causes a sympathetic response that only targets the nicotinic ACh receptors and AChE cannot degrade letting the muscles return to normal and bring the heart rate back to normal. B. The increased amount of ACh in her body is making it harder for her muscles to relax; therefore, there is a high concentration of Ca in her ECF and is having less cross bridges formed. C. The cause of an excess amount of ACh is because the voltage gated calcium channels on the terminal button are causing a parasympathetic response, which lead to contraction of the muscles and bronchoconstriction in smooth muscle. D. ACh is released from the preganglionic neuron in the thoracic region, which will bind to the muscarinic cholinergic receptors causing a parasympathetic response on the cell bodies of the postganglionic neuron. E. Mallory is experiencing a parasympathetic response; therefore, ACh is binding to muscarinic cholinergic receptors on the SA and AV nodes of the heart which is increasing her heart rate that is causing her shortness of breathe.

c

315. Which of the following statements is true about autonomic control of heart rate? A. Parasympathetic preganglionic neurotransmitter is released and binds to a muscarinic receptor which decreases the heart rate B. Sympathetic preganglionic neurotransmitter is epinephrine and binds to a adrenergic receptor which increases the heart rate C. Parasympathetic postganglionic neurotransmitter is acetylcholine and binds to a muscarinic receptor which decreases the heart rate D. Sympathetic postganglionic neurotransmitter is released and binds to muscarinic receptor which increases the heart rate E. Parasympathetic postganglionic neurotransmitter is norepinephrine and binds to a adrenergic receptor which decreases the heart rate

c

316. Of the following statements, which are true regarding skeletal muscles? A. Uses involuntary movement, controlled by autonomic nervous system through afferent motor neurons. B. Uses involuntary movement, controlled by somatic nervous system through afferent sensory neurons. C. Uses voluntary movement, controlled by somatic nervous system through efferent motor neurons. D. Uses voluntary movement, controlled by autonomic nervous system through afferent sensory neurons. E. Uses voluntary movement, controlled by somatic nervous system through efferent sensory neurons.

c

321. Lacey has been training UFC for more than 10 years and she will be fighting Tonya, who has an undefeated record. The fight began and punches are being thrown abruptly and it is a good fight. Lacey quickly threw her right fist to Tonya's left eye and blood begins to come down her face. Tonya's left eye begins to swell up in an instant and she falls to the ground. Then, Lacey is quickly in a choke-hold position with her arms, which are in a perpendicular position, while giving Tonya smaller amount of breaths to breathe. Tonya taps out and the referee checks to make sure if Tonya is alright while Lacey is running around the octagon with excitement for ending Tonya's undefeated career. What type of muscle contraction did Lacey use and how was Tonya's heart rate during the choke-hold position? a. Lacey used only an eccentric movement. Tonya's heart rate decreased because her blood flow decreased. b. Lacey used both isotonic and isometric movements. Tonya's heart rate increased because her blood flow increased. c. Lacey used both eccentric and concentric movement. Tonya's heart rate increase and her blood flow decreased. d. Lacey used only concentric movement. Tonya's heart rate increased and her blood flow increased. e. Lacey used only an isometric movement. Tonya's heart rate increased and her blood flow decreased.

c

326. Jessica feels that she is overweight and has tried to lose weight for years. One day she sees an ad for a weight loss surgery that could potentially help her lose the weight that she needs to. Jessica undergoes gastric bypass surgery and sees amazing results. Which of the following is true about the organ that underwent the surgery? A. This organ is lined with single unit smooth muscle that is innervated by the parasympathetic nervous system only and releases Ach from the post ganglionic neuron, which binds to a muscarinic cholinergic receptor on the effector organ. B. This organ is lined with multi-unit smooth muscle and is innervated by the parasympathetic nervous system only and releases Ach from the postganglionic neuron, which binds to a nicotinic cholinergic receptor on the effector organ. C. This organ is lined with single unit smooth muscle and is dually innervated. When innervated by the parasympathetic nervous system, Ach is released by the postganglionic neuron and binds to a muscarinic cholinergic receptor on the effector organ. When innervated by the sympathetic nervous system, norepinephrine is released by the postganglionic neuron and binds to an adrenergic receptor on the effector organ. D. This organ is lined with single unit smooth muscle that is innervated by the sympathetic nervous system only and releases Ach from the postganglionic neuron, which binds to a muscarinic cholinergic receptor on the effector organ E. This organ is lined with multi-unit smooth muscle that is dually innervated. When innervated by the parasympathetic nervous system, epinephrine is released by the postganglionic neuron and binds to an adrenergic receptor on the effector organ. When innervated by the sympathetic nervous system, Ach is released by the postganglionic neuron and binds to a nicotinic cholinergic receptor on the effector organ.

c

327. Michelle is a 19-year-old girl who experiences shortness of breath or dyspnea while doing physical activity. Her friends point out that her that she is fatigued often and her diet lacks a lot of iron and nutrients. After a few weeks, she goes to the doctor and discovers that her hematocrit is below normal and she is very Anemic. Which of the following are occur and contributing to Michelle's Anemia? A) Michelle has an increased concentration of hemoglobin in blood, therefore there is an increase in the amount of oxygen in the blood being delivered by Red Blood cells to the tissue. This leads to her having an obstructive disorder as she has too much oxygen in her system. B) Michelle has an increased concentration of red blood cells, therefore there is a decrease in the amount of oxygen in the blood being delivered by the Red Blood cells to the tissue. This makes her breath deeper and quicker in order to try to increase the amount of oxygen in the blood. C) Michelle has a deficiency of red blood cells, therefore there is a decrease in the amount of oxygen in the blood being delivered by the Red Blood cells to the tissue. This makes her breath deeper and quicker in order to try to increase the amount of oxygen in the blood. D) Michelle's shortness of breath during physical activity is due to her blood delivering an increased amount of oxygen the body requires. E) Michelle has an increased concentration of hemoglobin in blood, therefore there is an increase in the amount of oxygen in the blood being delivered by Red Blood cells to the tissue. This leads to her having an restrictive disorder as she does not have enough oxygen in her system.

c

341. Which of the following is true regarding insulin? A. Insulin is secreted by alpha cells of the pancreas, and is considered a catabolic hormone. B. Insulin is released in the post-absorptive state and works to prevent hypoglycemia. C. Insulin is released by beta cells in the pancreas, and dominates in the absorptive state. D. Insulin lowers blood glucose levels by increasing glycogenolysis, gluconeogenesis, and ketogenesis. E. Insulin is considered an anabolic hormone and is released when blood glucose levels are low.

c

343. Which of the following statements is false about potassium? A. During excitation-contraction coupling, when acetylcholine binds to nicotinic cholinergic receptors on the motor end plate, potassium ions move out of the cell as sodium ions move into the cell. B. Potassium efflux occurs when voltage-gated potassium channels open during a cardiac action potential, which causes a brief period of repolarization. C. Potassium is used during the death penalty because when potassium floods the heart, it causes the heart rate to decrease and eventually stop if enough potassium is applied. This is due build up of negative charged ions in the cell. D. During an action potential, if there is more potassium efflux, repolarization occurs for a longer time, and it would take longer for the membrane potential to reach threshold again. E. During a parasympathetic response, when acetylcholine binds its receptor protein, direct coupling occurs which augments voltage-gated calcium T-type channels and inhibits voltage-gated potassium channels.

c

358. Jo Bob Jr the 4th is 17 years old and finds himself feeling weak and tired, has to urinate frequently and he cannot quench his thirst. No insulin is being secreted from his pancreas. Which of the following is true for Jo? A. He has an excessive number of Beta cells in the pancreas. B. The ligand that aids in bringing glucose into the cell has too low of an affinity to its receptors resulting in a high number of glucose in the blood. C. He has a dangerously high number of ketones in his blood. D. His rate of glycolysis is too slow, so he is not producing enough ATP to give him energy. E. His muscarinic cholinergic receptors are not functioning properly.

c

362. You are stressed out and cramming for finals, causing your body to release excess amounts of cortisol. Which statement is TRUE about cortisol? a. It is a lipophobic steroid hormone that derived from cholesterol, which requires a transmembrane protein in order to enter your cells. b. It is released when ACTH binds to receptors in your zona glomerulosa. c. It increases the concentration of glucose, ketone bodies, amino acids, and fatty acids in your blood plasma. d. It is secreted during a parasympathetic response and binds to muscarinic cholinergic receptors. e. Prolonged release of cortisol will act on your thymus to increase your immune response, preventing you from getting sick.

c

378. Which statement is true if ferritin (protein in mucosal cells of duodenum that stores/absorbs Fe) was denatured? a. Body's hematocrit levels will increase. b. Cellular tissues throughout the body will have increased levels of oxygen supplied. c. Anemic symptoms will occur. d. Water absorption in intestinal lumen will be inverted. e. Kidney will increase absorption of ions in descending part of Loop of Henle.

c

255. Grant is taken to the emergency room because he is complaining of a lot of chest pain. It is possible that the cause of Grant's pain is... a. Grant has asthma, a restrictive lung disease in which the lungs cannot expand to their fullest, resulting in Grant feeling a tight, painful chest and not being able to breath well. b. Grant is having a pneumothorax, and air is entering his lungs at a fast rate and his lungs are expanding more than they should, so he is feeling a lot of pressure inside his chest. c. Grant's CVP is too high, so venous return is decreased and his heart is not receiving enough blood, so the heart is having to pump much harder, which creates pain in the chest. d. Grant is having an anxiety attack, so large amounts of Epinephrine are being released in the body. His heartbeat has increased and become somewhat irregular and Epinephrine is causing Grant to experience shortness of breath, resulting in chest pain. e. Grant was sleeping so the brain, a reconditioning organ, was not in need of as much oxygenated blood, so his heart rate slowed down and has not recovered, so blood is not being delivered back to his myocardium fast enough.

d

256. Which of the following is correct about potassium in the body? a. A low potassium level in the hair cells of the ear make more action potentials occur. b. A low potassium level in the hair cells of the ear make more action potentials occur. c. A high potassium level in the hair cells of the ear make less action potentials occur. d. A low potassium level is outside of a heart cell. e. A high potassium level is outside of a heart cell.

d

260. Which of the following is NOT true about the sympathetic system? a. It uses nicotinic cholinergic receptors. b. The sympathetic nervous system triggers an increase in heart rate, dilated pupils, and constricts blood vessels. c. The release of the preganglionic sympathetic neuron, Acetylcholine, triggers the release of epinephrine and norepinephrine. d. The only receptors that the neurotransmitters of the sympathetic system can bind to are alpha one and beta one. e. It is a part of the autonomic nervous system.

d

261. You are a clinical rotation student who is working on your emergency room rotation at Norman Regional Hospital. You have a female patient that was just brought in from a car accident that has faced several injuries resulting in a great deal of blood loss. Which is the most likely to occur? a. She will be fine when the doctors stitch her up, because the blood will be locked in her body. b. She likely has low blood pressure, which will send signals to the brain via the efferent pathway to the hypothalamus. This will then signal the release of luteinizing hormone, which will help her body to restore some of the blood volume lost. c. Her blood pressure will increase sending more blood to the injured parts of the body, this will send signals to the brain signaling the release of ADH, which is a hormone that will help lower the blood pressure by acting as a diuretic and lowering blood volume. d. Her blood pressure will decrease, sending signals to the brain via the afferent pathway to the hypothalamus, which will stimulate the posterior pituitary to release ADH. This will help maintain blood volume by forcing the kidneys to reabsorb water, and halt urine production. e. Her blood pressure will increase, because her open wounds have changed the pressure gradient so as there will be higher pressure inside the body, and lower pressure outside the body. This will force more blood out of her body until the gradient is equalized or she is stitched up.

d

265. Which of the following statements is true about the second messenger system cAMP? a. Acetylcholine uses the cAMP second messenger system in a pacemaker cell to activate channels to generate a slower heart rate. b. All water-soluble hormones can freely diffuse across a cell membrane and therefore do not have to use the cAMP second messenger system. c. cAMP causes the release of calcium ions from the cytosol of a cell. d. Epinephrine uses the cAMP second messenger system which activates protein kinase A to phosphorylate channels to increase heart rate. e. In the second messenger system of cAMP, phosphodiestrase breaks down the protein kinase.

d

278. Travis stayed up all night partying with his friends. On Monday morning, he decided to order a triple shot carmel macchiato from Starbucks to keep him awake. Explain how the caffeine in the coffee keeps Travis from feeling tired that day. a. The caffeine in the coffee acted as an antagonist for acetylcholine, inhibiting the muscarinic cholinergic receptors on the cardiac muscle of the heart. b. The caffeine in the coffee acted as an agonist of epinephrine, binding to the beta adernergic receptors on the cardiac muscles of the heart. c. The caffeine in the coffee blocks adenylate cyclase in the cAMP second messenger system, causing a decrease in the breaking down of epinephrine. d. The caffeine in the coffee blocks phosphodiesterase in the cAMP second messenger system causing an increase of the effects of epinephrine. e. The caffeine in the coffee blocks the protein kinase in the cAMP second messenger system, which allows a greater influx of Na+ ions, and increases the effects of epinephrine.

d

280. Julie schedules a visit with her doctor after experiencing extreme thirst, dehydration, and frequent headaches. Her doctor tells her she has developed an inactive tumor on her hypothalamus and diagnoses her with Diabetes Insipidus. How is this inactive tumor affecting Julie's hormone levels and consequently her blood pressure? (NO ANSWER GIVEN) a. Hypersecretion of ADH from paraventricular nuclei in the posterior pituitary; too much ADH to bind to receptors on the kidneys, so these receptors down-regulate, ultimately leading to less sodium reabsorption, less water conservation, and a decrease in blood pressure. b. Hyposecretion of ADH from the supraoptic nuclei in the hypothalamus; too little ADH to bind to receptors on the kidneys, leading to less calcium reabsorption, less water conservation, and a decrease in blood pressure. c. Hyposecretion of insulin from beta cells in the pancreas; not enough insulin binding to receptors on the liver, leading to decreased levels of glucose in the liver and increased glucose levels in the blood, causing an increased blood volume and an increased blood pressure. d. Hyposecretion of ADH from paraventricular nuclei in the hypothalamus; too little ADH binding to receptors on the kidneys, so now the kidneys are unable to respond by increasing water and sodium absorption, which will lead to a decrease in blood volume and a decrease in venous return, ultimately lowering blood pressure. e. Hypersecretion of insulin from beta cells in the pancreas; excess of insulin causes down-regulation of receptors on the liver, leading to abnormally high A1C levels and an increase in blood volume, therefore increasing blood pressure.

d

290. Which of the following is true about acetylcholine? a. In arteriolar smooth muscle, acetylcholine binds to α1 cholinergic receptors, which activate the cAMP second messenger system. This causes vasoconstriction of the arterioles which will decrease the total peripheral resistance (TPR) and increase the mean arterial pressure (MAP), which will eventually cause less blood flow to vascular smooth muscle in the body. b. In the parasympathetic nervous system, acetylcholine is released from the preganglionic fiber and binds to muscarinic cholinergic receptors on the cell body and dendrites of the postganglionic fiber. c. In the frog heart lab, acetylcholine was placed on the frog's heart causing a sympathetic response. This occurred because acetylcholine binds to β2 adrenergic receptors on the heart, causing more influx of Na and Ca and efflux of K. Because of this, you see a decreased period of depolarization and a decreased repolarization period causing the waves to be shorter and closer to threshold. d. In the somatic nervous system, acetylcholine is released at the neuromuscular junction from a motor neuron and binds to nicotinic cholinergic receptors on skeletal muscle. Nicotinic cholinergic receptors trigger a fast response causing Na+ influx and K+ efflux. Na+ influx causes an end plate potential to occur on the motor end plate which triggers an action potential to travel down the sarcolemma eventually causing muscle contraction. e. During a sympathetic response, Epinephrine is released from the post-ganglionic fiber and binds to an adrengenic receptor, while Acetylcholine is released from the pre-ganglionic fiber and binds to a nicotinic cholinergic receptor which generates a slow response.

d

303. When Dr. Ketchum injured her knee during a competition riding her horse, she was rushed to the hospital. While in the emergency room, she learned that her knee was worse than she thought it was. She soon found out she would have to have knee replacement surgery. As she was going into surgery, she realized that one of the nurses was one of her former TA's. Dr. Ketchum and her former TA began to chit chat, and while they were talking the nurse placed an IV in Dr. Ketchum's arm, and the IV began to drip. A minute or two later, the nurse realized that she gave Dr. Ketchum the wrong IV; instead of 0.9% NaCl, she gave Dr. Ketchum an IV containing 9.0% NaCl! In that short amount of time, what most likely happened to Dr. Ketchum due to the increase of Sodium? A. Dr. Ketchum's heart rate would increase; the increase of Na affected Dr. Ketchum's heart, causing an even shorter slow depolarization in a cardiac contractile cell, causing her heart rate to increase. B. More Excitation Contraction Coupling will occur in Dr. Ketchum's muscle cells, due to more Na+ binding to troponin, shifting tropomyosin over to expose the myosin binding sites, causing more contraction of her muscle fibers. C. There will be an increase of activity in the Sodium potassium pumps, because more Na+ efflux will cause more K+ influx, causing Dr. Ketchum's Sodium/Potassium pumps to function faster. D. The increase in voltage causes more sodium influx into the ICF through the voltage gated sodium channels on the axon hillock of Dr. Ketchum's neurons, causing a change in the membrane potential in her neurons, thus increasing the chances of action potentials being produced. E. The increase in sodium will created a longer plateau in the cardiac contractile cells in Dr. Ketchum's heart, due to the influx of sodium and efflux of potassium at the same time during a heart contraction.

d

308. Which of the following statements are false if the production of ACh in the body stopped and an ACh agonist could not be administered? A. Preganglionic neurons of the parasympathetic and sympathetic nervous systems could not communicate via autonomic ganglion with the post ganglionic neurons B. The skeletal muscles will not contract because no ACh is will bind to nicotinic cholinergic receptors to allow Na and K ion movement to make and end plate potential. C. Blood pressure would decrease due to no vasoconstriction from the sympathetic nervous system. D. There would be excessive bronchoconstriction in the lungs do to the lack of ACh binding to muscarinic cholinergic receptors in the smooth muscles. E. There would be no pupil constriction do to no ACh binding

d

309. 73-year-old Josiah Biles was diagnosed with Parkinson's disease, (a degenerative disorder due to death of Dopamine producing cells within the Substantia Nigra in the midbrain) after going to the doctors with complaints of shakiness, slowness of movement and problems with his gait and signs of depression. Josiah is confused and extremely distraught, he is wanting to better understand the effects of this disease on his body, please help Josiah by choosing the statement that is FALSE regarding Parkinson's Disease. A. Due to the astrocytes and tight junctions forming the blood brain barrier, Dopamine is unable to cross directly into the central nervous system. In order to treat this disease, he must be proscribed L-Dopa, a dopamine agonist. B. He has problems with voluntary motor function, thus affecting the somatic nervous system. The somatic nervous system consists of skeletal muscle. Skeletal muscle contraction includes cross bridge cycling initiated by the binding of Calcium to Troponin. C. Josiah has experienced death of Dopamine producing cells within the midbrain. Which may very well explain non-motor difficulties he may have such as problems sleeping. D. L-Dopa medication acts on your sympathetic nervous system. During a sympathetic response, Acetyl Choline binds to Beta 1 adrenergic receptors located on the heart and will lead to an increased heart rate. E. Dopamine is a Lipophobic Biogenic Amine and therefore does not cross the blood brain barrier. Degradation of Biogenic Amines such as Dopamine and serotonin requires the use of Mono-amine Oxidase.

d

313. Which of the following statements below is FALSE? A. The conductance System is: SA node, AV node, Bundle of His, Left and Right Bundle Branches, and purkinje fibers. B. Calcium influx must occur, entering the preganglionic cell to release ACh to travel across the synaptic cleft and bind to nicotinic cholinergic receptors on the postganglionic cell. C. The pulmonary artery carries deoxygenated blood into the lungs, when loading of oxygen and unloading of carbon dioxide occur in the capillaries. D. The sympathetic division releases Epinephrine and Norepinephrine which bind to adrenergic receptors on circular muscle. E. The adrenal medulla has preganglionic fibers, but does not have postganglionic fibers, instead they are referred to as chromaffin cells, which release mostly epinephrine into the bloodstream.

d

318. Which of the following statements is true regarding phototransduction with the absence of a light stimulus? A. When there is no light stimulus, a fast IPSP is generated on the bipolar cell as a result of glutamate binding to inotropic receptors. B. High levels of cGMP activate Na channels to open, hyperpolarizing the rod. Ca channels are then triggered to open, causing glutamate to be released in the synaptic cleft. C. In the absence of light rhodopsin disassociates, which activates transducin. This then activates phosphodiesterase, which raises the levels of cGMP in the cell. D. After glutamate binds to receptors on the bipolar cell, an IPSP graded potential is generated, decreasing the likelihood of an action potential being sent to the visual cortex of the brain. E. The neurotransmitter released in the synaptic cleft is an excitatory biogenic amine.

d

320. In patients with Type 2 Diabetes, the drug Metformin, is used to treat hyperglycemia by decreasing gluconeogenesis. This is done by Metformin acting as a Glucagon antagonist, it inhibits adenylate cyclase which lowers the effectivity of Glucagon. What mechanism is used to allow Metformin to suppress gluconeogenesis? A. Glucagon is a lipophilic hormone which means it will bind to receptors on the membrane and work through the second messenger system. Metformin inhibits adenylate cyclase, thus, cGMP will not be able to activate Protein Kinase A, and glucagon will be less effective in promoting gluconeogenesis in the liver. This will then lower blood glucose levels. B. Glucagon is a lipophobic hormone which means it will bind to receptors on the membrane and work through direct coupling. Metformin inhibits adenylate cyclase, thus, cAMP will not be able to activate Protein Kinase A, and glucagon will be less effective in promoting gluconeogenesis in the liver. This will then lower blood glucose levels. C. Glucagon is a lipophobic hormone which means it will bind to receptors in the cell and work through second messenger system. Metformin inhibits adenylate cyclase, thus, cAMP will not be able to activate Protein Kinase A, and glucagon will be less effective in promoting gluconeogenesis in the liver. This will then lower blood glucose levels. D. Glucagon is a lipophobic hormone which means it will bind to receptors on the membrane and work through second messenger system. Metformin inhibits adenylate cyclase, thus, cAMP will not be able to activate Protein Kinase A, and glucagon will be less effective in promoting gluconeogenesis in the liver. This will then lower blood glucose levels. E. Glucagon is a lipophobic hormone which means it will bind to receptors on the membrane and work through the second messenger system. Metformin inhibits adenylate cyclase, thus, cAMP will not be able to activate Protein Kinase A, and glucagon will be less effective in promoting gluconeogenesis in skeletal muscle. This will then lower blood glucose levels.

d

324. Which of the following statements is true regarding the respiratory zone within the respiratory system? A. The respiratory zone begins where the primary bronchi meets the terminal bronchioles within the lungs. B. The respiratory zone functions as a place for gas exchange in order to create concentration gradients that allow CO2 to use primary active transport more efficiently because there is more CO2 within the lungs than within the body. C. The main purpose of the respiratory zone is to provide pathways for oxygen to enter the lungs. D. Gas exchange takes place within the respiratory zone when oxygen reaches the alveoli, where passive transport allows oxygen to diffuse into the cells. E. Alveoli within the respiratory zone allow oxygen to diffuse into the cell using active transport because there is more oxygen within the alveoli than outside.

d

325. Sally contracted a rare unknown disease in which her body stopped being able to utilize any second messenger systems. Which of the following statements is FALSE about what could happen to Sally? A. Sally would not be able to vasoconstrict her arterioles. B. Sally would not be able to increase the force of contractility of her heart due to protein kinase A not being activated. C. Sally would have increased urination due to the inability of ADH to function using a second messenger. D. Normal cross bridge cycling could not occur due to second messenger systems not being able to open ion channels on the sarcolemma. E. There would be no parasympathetic responses because of the inability of muscarinic cholinergic receptors at the postsynaptic neurons of the parasympathetic nervous system to use the second messenger system.

d

330. The drug Vyvanse is used to treat attention-deficit hyperactive disorder (ADHD) and is classified as a central nervous system stimulant. Vyvnase works by blocking the reuptake molecule on the axon terminal, increasing the amount of dopamine and norephinephrine in the synaptic cleft available to bind to receptors on the presynaptic neuron. Based on your knowledge of neurotransmitters and their characteristics, what effect will this drug have on the body? A. Norepinephrine will bind to muscarinic cholinergic receptors on the SA and AV nodes in the heart, causing an increase in heart rate. Norepinephrine will also bind to muscarinic cholinergic receptors in the iris causing the pupils to dilate. This is a sympathetic nervous system response. HE WOULD HAVE TO REPHRASE THIS B. Norepinephrine will bind to muscarinic cholinergic receptors on the SA and AV nodes in the heart, causing a decrease in heart rate. Norepinephrine will also bind to muscarinic cholinergic receptors in the iris cause the pupils to constrict. This is a parasympathetic nervous system response. C. Norepinephrine will bind to muscarinic cholinergic receptors on the SA and AV nodes in the heart, causing an increase in heart rate. Norephinephrine will also bind to muscarinic cholinergic receptors in the iris causing the pupils to constrict. This is a sympathetic nervous system response. D. Norepinephrine will bind to nicotinic cholinergic receptors on the SA node, AV node, and ventricular myocardium in the heart, causing an increase in the heart rate. Norepinephrine will also bind to nicotinic cholinergic receptors in the iris causing the pupils to dilate. This is a sympathetic nervous system response. E. Norepinephrine will bind to nicotinic cholinergic receptors on the SA node, AV node, and ventricular myocardium in the heart, causing a decrease in heart rate. Norepinephrine will also bind to nicotinic cholinergic receptors in the iris causing the pupils to dilate. This is a sympathetic nervous system response.

d

342. Dave is a student TA at the University of Oklahoma who is covering a lecture for Dr. Ketchum while she is out of town competing with her horses. This is the first lecture Dave has ever taught and he is beginning to feel nervous. His hands begin to sweat and his breathing rate begins to increase along with his heart rate. Which circuit of the autonomic nervous system is dominating, and which hormone(s) release is causing these side effects? A. The sympathetic nervous system and the hormone acetylcholine B. The parasympathetic nervous system and the hormone acetylcholine C. The sympathetic nervous system and the hormones epinephrine and acetylcholine D. The sympathetic nervous system and the hormones epinephrine and norepinephrine E. The parasympathetic nervous system and the hormones epinephrine and norepinephrine

d

356. The day after thanksgiving, Maddy woke up with her chest burning and it felt like her throat was closing up. Her cousin took her to the doctor, but while waiting in the ER, she began struggling to exhale, causing her chest and throat to burn more. Maddy has a history of smoking. What did the doctor diagnose her with? A. Maddy has emphysema, a restrictive lung disorder which limits her gas exchange. B. Maddy has a chronic obstructive pulmonary disorder that causes the blood level in her lungs to exceed the normal 15% of blood in the body. C. Maddy's oxygenated blood that is supposed to flows from by means of the pulmonary artery to the lungs is unable to get to the lungs due to artherosclerosis in her arteries. D. The doctor diagnosed Maddy with emphysema, which damaged the inner walls of her air sacs causing them to rupture. This created one large air space preventing efficient gas exchange from taking place. E. The oxygenated pulmonary vein used to transport blood from the lungs to the heart constricted due to the damage from the smoke inhaled during Maddy's previous years. This caused her blood flow to be restricted, in turn creating a lower blood pressure. This low blood pressure prevented the blood from pumping all the way through the body and back to the lungs. In turn, the doctor diagnosed Maddy with hypotension.

d

357. Michelle has been under a lot of stress since starting OU Nursing School this semester, but luckily since she was once Dr. Ketchum's student, she thinks she remembers what's occurring in her body. Since it's been a while since she last took Physiology, she's not sure which one of her statements regarding cortisol applies to her life right now. Help Michelle out! Which one of the following statements regarding Cortisol is true? A. Cortisol is a lipophobic glucocorticoid that is synthesized by the zona fasiculata. B. Due to the hypersecretion of cortisol in Michelle's bloodstream, negative feedback will cause ACTH levels to increase and CRH levels to decrease. C. Cortisol is a hormone that targets the liver, which will then cause Glycogeneogenesis to occur. D. The receptor for Cortisol is inside the target cell, in which will produce an intracellular response, including lipolysis in the adipose tissue and proteolysis in the skeletal muscle. E. The adrenal cortex releases Cortisol in the bloodstream where it will then dissolve as it's transported in the blood.

d

264. Dr. McDreamy was looking through a patient's chart. He noticed the patient had been given an injection of Atropine, but the intern forgot to write down why it was needed. Which of the following could have been the reason for the injection? a. The patient needed an anticholinergic to block g-protein coupling of the nicotinic receptors in the eyes which will cause the pupils to dilate. b. The patient needed an anticholinergic to block g-protein coupling of the muscarinic receptors in the eyes which will cause the pupils to constrict. c. The patient's heart was racing, so the injection caused a sympathetic response by decreasing pace maker potentials from the SA node in the left Atrium. d. The patient's heart rate was below normal, so the injection was used for a parasympathetic response to increase pace maker potentials from the SA node in the right Atrium. e. The patient's heart rate was below normal, so the injection was used to counteract parasympathetic responses and increase pace maker potentials from the SA node in the right Atrium.

e

360. A male college student was admitted to the emergency room and diagnosed with severely elevated heart rate and blood pressure levels. These levels remained high, even after the student spent a period at rest. The student claims that he had been lifting very heavy weights in the gym when he was suddenly overcome with a severe headache, significant skin flushing, and the inability to catch his breath. He also claimed that, even after attempting to relax, his heart would not stop beating at an alarming rate. Which of the following diagnoses is most likely to be the cause of this health issue? A. By overexerting himself while weightlifting, the student sustained damage to his sinoatrial node, causing his atrioventricular node- which produces a faster pacemaker potential- to take control of his heartrate. B. The student suffers from a genetic condition that causes hypersecretion of adrenaline in response to physical exertion. These high levels of adrenaline then bind to the Beta-1 muscarinic receptors in the heart, causing a dangerous increase in heart rate. C. The student consumed an unhealthy amount of caffeine prior to visiting the gym, which caused his body to undergo a severely elevated parasympathetic response upon physical exertion. D. Because the student suffers from damaged vagus and glossopharyngeal nerves, his baroreceptors cannot properly inform his medulla of the increase in blood pressure brought on by exercise, preventing his body from properly decreasing sympathetic and increasing parasympathetic responses to lower blood pressure levels. E. The student suffers from a genetic condition that inhibits his ability to produce sufficient quantities of norepinephrine and epinephrine, preventing his body from properly undergoing a parasympathetic response after exercising.

d

365. Which of the following statements is FALSE about the characteristics of cardiac and skeletal muscle? a. The cardiac muscle duration of absolute refractory period in cardiac muscle is longer than skeletal muscle. b. In cardiac and skeletal muscle, Calcium binds to troponin and initiates muscle contraction. c. The nerve impulse creates an action potential in skeletal muscle while the cardiac muscle action potential produces pacemaker cells. d. Cardiac muscle's action potential stimulates DHP receptors on T Tubules and does not cause an influx of calcium into the muscle cell. e. Cardiac muscle is myogenic and skeletal muscle is neurogenic.

d

366. 65-year-old Daveed has just woken up from his general anesthetic after having surgery on a hernia in his abdomen. Daveed complains of intense pain in his abdomen where the incision was made for his surgery, and therefore refuses to move around or take deep breaths in fear of contracting his abdominal muscles. Daveed is slightly overweight, and has been smoking two packs of cigarettes everyday for the past nineteen years, and therefore is suffering from onset chronic obstructive pulmonary disease. Which of the following is Daveed NOT at risk for? a. The patient's inactivity can cause blood to become trapped in the lungs because the heart is not pumping the blood properly. This trapped blood in the lungs can cause the capillaries to leak and lead to accumulation of fluid in the lungs b. The patient's history of obesity and smoking and his immobility causing poor circulation of blood, can lead to blood clots in his lower extremities. c. The patient's refusal to take deep breaths on top of his onset COPD can cause air to become trapped in the lungs, which can lead to the buildup of CO2, and can ultimately lead to respiratory distress. d. The patient is at risk for "going back under" due to the trace amounts of anesthetic still in his blood not being exhaled, blocking receptors of the central nervous system. e. The patient can develop a blockage of his G.I. tract due to his bowels not waking up from the anesthesia and the patient's refusal to get up and walk around.

d

367. Kaitlin was doing the frog heart lab, when she accidently switched her water bottle for the bottle containing Ach, so she took a big drink of the bottle containing Ach (She didn't follow lab safety rules very well). Because of this mistake, her levels of Ach were above normal. What would NOT happen due to the elevated Ach levels? a. Post ganglionic receptors would down regulate due to the high levels of Ach b. Skeletal muscles would begin to contract c. Ach would be released from preganglionic neurons from the sacral region of the spine d. Parasympathetic response in the ventricular myocardium e. AchE would be inhibited

d

369. It was a Wednesday night after lab when Reid walks into a bar and runs into his friend Elizabeth. "How are you?" Elizabeth asks. "Actually, not great," answers Reid. Reid then tells Elizabeth that lately he has been losing weight, has muscle weakness, and his heart rate has been extremely high. Elizabeth takes Reid to the doctor, and the doctor says that Reid has a functioning adenoma on his thyroid gland. Which of the following is NOT true regarding Reid's current state? a. The functioning adenoma is causing hypersecretion of calcitonin, causing reduced blood calcium levels. The reduced calcium levels are resulting in less calcium being available to bind to troponin, which would allow for tropomyosin to shift over, exposing myosin binding sites. Therefor, less calcium binding to troponin is causing less cross-bridge cycling, resulting in decreased ability for muscle contraction. b. The functioning adenoma is causing hypersecretion of thyroid hormone, which is causing a higher frequency of cardiac action potentials, which is increasing heart rate. This increase in heart rate causes an increase in cardiac output and therefor an increase in blood pressure. c. The functioning adenoma would cause hypersecretion of thyroid hormone which is increasing his metabolic rate. This is causing chemical digestion of large molecules (such as carbohydrates, lipids, proteins, and nucleic acids) by enzymes through hydrolysis to occur much faster. Then they can be absorbed by the lining of the alimentary canal quicker too. d. Reid could be classified as having hypothyroidism, caused by an elevated level of thyroid hormone in the blood. e. Reid would have low levels of TRH and TSH due to negative feedback.

d

371. Which of the following is true about vasopressin (ADH)? a. It is a peptide hormone synthesized and secreted by the anterior pituitary. b. The release of vasopressin increases blood volume and decreases blood pressure through a positive feedback loop. c. It is also known as Adrenocorticotropic Hormone, targets the kidneys and increases water absorption. d. The release of vasopressin increases blood volume and increases Mean Arterial Pressure (MAP) through long-term regulation. e. Alcohol hangovers are caused by the increased release of vasopressin.

d

380. Heather was walking home from the Bizzell Memorial Library alone after a long night of studying for her physiology class when suddenly she noticed a dark figure following her. She immediately felt a surge go through her body as her sympathetic nervous system responded to the situation. However, as she began to run, she felt extremely fatigued. Luckily the dark figure disappeared, yet the situation left Heather feeling extremely concerned about her health. She knew from her studying that she should not have felt fatigued during a fight or flight response. The next day, she visited the Goddard Health Center and explained what had happened. Her physician ran a multitude of tests to discover what the problem might be. What issue did the physician at Goddard find with Heather's health? a. Heather's EKG showed multiple P waves per QRS complexes. She was diagnosed with a complete heart block due to the inability of her SA node to conduct electrical impulses. Therefore, her fatigue was caused by the inability of her sympathetic nervous system to increase her heart rate and the delivery of oxygen to her muscles. b. Heather's spirometry test revealed that she had asthma, a restrictive pulmonary disease that prevented the bronchodilation that should have occurred during her fight or flight response. Therefore, her fatigue was caused by an insufficient amount of oxygen being brought into the body during inspiration. c. Following a CT scan, the physician discovered a non-functioning adenoma on Heather's adrenal cortex. Therefore, her fatigue was caused by the inability of the adrenal cortex to secrete the catecholamines necessary for the sympathetic response. d. Heather's physician performed a muscle biopsy which showed that she had a mitochondrial disease. Upon further testing, the specific issue was found to be a genetic mutation that caused Heather to have malfunctioning ATP synthase enzymes on her inner mitochondrial membrane. Therefore, her fatigue was caused by her inability to perform chemiosmotic coupling during aerobic respiration, forcing her to rely on anaerobic respiration which produces much less ATP. e. Further genetic testing showed a genetic mutation regarding Heather's tropomyosin protein. This mutation altered the binding site on tropomyosin making it very difficult for calcium to bind. With less calcium binding to tropomyosin, there was less shifting of troponin, and therefore less exposing of myosin binding sites on the actin filaments. Therefore, Heather's fatigue was caused by a decreased ability for actin and myosin to perform the power stroke, decreasing the amount of contraction that could occur in her skeletal muscles.

d

286. Lindsay was playing in a soccer game and she was working herself a little too much. All of a sudden Lindsay slowed down and there was a dramatic decrease in her heart rate. Which of the following could be true in regards to causing her heart rate to decrease? a. The sympathetic NS is activated causing an decrease in your heart rate due to the neurotransmitter binding to the muscarinic cholinergic receptors. b. Slow depolarization occurring allowing the heart to an extended influx of potassium. c. The vagus nerve is innervating the SA and AV nodes, which are located in the left atrium, more rapidly than normal causing a lower frequency of action potentials. d. An abundance of epinephrine and norepinephrine are released from the postganglionic neuron and bind to β1 adrenergic on the effector organ. e. The cells of the SA nodes aren't functioning allowing the AV to take over the heart rate. The AV node cannot function as fast as the SA node causing a lower frequency of action potentials to be generated, which in turn causes a decrease in heart rate.

e

288. Which of these statements is true regarding the addition of Pilocarpine to the heart? a. Pilocarpine is an Acetylcholine agonist and binds to muscarinic cholinergic receptors; pilocarpine will increase the frequency of pacemaker potentials, which will cause an increase in heart rate. b. Pilocarpine is an Acetylcholine agonist and binds to nicotinic cholinergic receptors; pilocarpine will increase the frequency of pacemaker potentials, which will cause an increase in heart rate. c. Pilocarpine is an Acetylcholine antagonist and binds to muscarinic cholinergic receptors; pilocarpine will decrease the frequency of pacemaker potentials, which will cause a decrease in heart rate. d. Pilocarpine is an Acetylcholine antagonist and binds to adrenergic receptors; pilocarpine will decrease the frequency of pacemaker potentials, which will cause a decrease in heart rate. e. Pilocarpine is an Acetylcholine agonist and binds to muscarinic cholinergic receptors; pilocarpine will decrease the frequency of pacemaker action potentials, which causes a decrease in heart rate.

e

294. When long-distance runners compete they run for a long period of time over a very long distance. Sprinters on the other hand run very quick short distances. Which of the following are correct about their muscle fibers? A. Sprinters will have larger muscles due to slow glycolytic muscle fibers and long-distance runners will have smaller muscles due to the small diameter of slow oxidative fibers. B. Slow oxidative muscle fibers in long-distance runners have a larger diameter than the fibers in sprinters. C. Sprinters muscle fibers have a larger diameter because they use mainly use aerobic respiration. D. Long-distance runners have more fast glycolytic because they need more energy for a longer period of time. E. The short distance that sprinters run cause recruitment of muscle fibers with a larger diameter and use anaerobic respiration.

e

302. Brittany had a traumatic car accident in which her car was hit head on by a large truck. Most of her body was unharmed except for a puncture through her chest, which caused an increased shortness of breath. In addition to this, she was very alert and energized from all of the stress that was placed on her during her traumatic experience. What trauma might have caused Brittany's shortness of breath and what hormone was released in response to this situation? A. Brittany most likely has a pneumothorax that resulted in her increased shortness of breath, because her intra-alveolar pressure was higher than her atmospheric pressure. The hormone that was released was epinephrine, which increased her alertness and energy levels. B. Brittany most likely has increased shortness of breath due to chronic bronchitis, which caused her airways to become narrow. The hormone that was released was aldosterone to help retain water for more energy. C. Brittany most likely has a pneumothorax that resulted in her increased shortness of breath, because her intra-pleural pressure was equal to her atmospheric pressure. The hormone that was released was acetylcholine, which increased her alertness and energy levels. D. Brittany most likely has a pneumothorax that resulted in her increased shortness of breath, because her intra-pleural pressure was higher than her atmospheric pressure. The hormone that was released was epinephrine, which increased her alertness and energy levels. E. Brittany most likely has a pneumothorax that resulted in her increased shortness of breath, because her intra-pleural pressure was equal to her atmospheric pressure. The hormone that was released was epinephrine, which increased her alertness and energy levels.

e

311. On Halloween, you decide to go to a haunted house. Before entering the house you realize your palms are sweaty, your heart is racing, and you are incredibly nervous. Which of the following statements is NOT true? A. the sympathetic nervous system is taking over B. you would have faster repolarization of the ventricles C. the lubb/ dubb sounds would be closer together D. epinephrine would be released and bind to adrenergic receptors on target tissues E. acetylcholine would be released and bind to muscarinic cholinergic receptors on target tissues

e

312. Which of these functions is not true of the the adrenal medulla? A. It produces messengers for the sympathetic nervous system B. It helps a person cope with physical and emotional stress C. It does not secrete aldosterone which helps maintain blood volume and pressure D. The amygdala alarms the hypothalamus to eventually stimulate this organ E. Its products allow for intrinsic control of vasodilation to deliver blood to reconditioning organs

e

323. Amber is a smoker who is unaware of the effects of nicotine on her body. Which of the following would be FALSE to tell Amber about nicotine's role in her body? A. Nicotine acts as a sympathomimetric drug at preganglionic synapses causing the postganglionic fiber to release norepinephrine onto the target organ. B. In the parasympathetic nervous system nicotine mimics acetylcholine at preganglionic synapses causing postganglionic fibers to release acetylcholine onto the target organ. C. Amber is likely to get an obstructive lung disorder from her extensive smoking which would decrease her ERV. D. Amber is likely to get an obstructive lung disorder from her extensive smoking which would decrease her FVC. E. The norepinephrine and acetylcholine released by postganglionic fibers of the sympathetic and parasympathetic nervous systems (respectively), which were caused by nicotine mimicking acetylcholine, would cancel each other out.

e

363. Granny Goose fell 2 months ago while she was working at her desk at work. She remembered perfectly everything that happened and claimed she had lost her footing. She received a laceration on her eyebrow about ½ of an inch deep and 2 inches long. After she went to the emergency room, she was given 36 stitches and went home. Today, she claims she has numbness that extends from her eyebrow to the back of her head. She also has muscle impairment in the area. Which of these answer choices is NOT a possible result from Granny Goose's fall? a. Granny Goose cut the nerve fibers in the area, which resulted in lack of communication to the brain from the nerve fibers. Therefore the skeletal muscle in the face no longer responded causing numbness in the area. b. Calcium influx no longer occurred causing the muscle in the eyebrow and forehead to decrease the amount of cross bridge cycling that occurred which in turn decreased contraction, causing the numbness. c. Acetylcholinesterase is over stimulated in the area. d. The neurons can no longer reach threshold. Action potentials can no longer propagate down the axon from the axon hillock effectively. e. IP3 can no longer be activated to produce a response in the skeletal muscle to move.

e

379. Which of the following is true about the respiratory system? a. Air travels first through the oral cavity, then the trachea, pharynx, primary bronchi, and finally the lungs b. The major entrance/exit for the respiratory system is through the mouth c. The nasal cavity is composed mostly of alveolar cells, which are epithelial cells used that produce mucus that help trap foreign debris d. The pulmonary artery originates in the pulmonary trunk and carries oxygenated blood to the alveoli e. The parasympathetic NS causes bronchoconstriction, while the sympathetic NS causes bronchodilation

e

381. Which of the statements is true regarding acetylcholine? a. It is a hormone released by the parasympathetic nervous system during times of "resting and digesting". b. Its actions would be inhibited by the addition of a nicotinic agonist. c. It is released by the nervous system to activate motor neurons. d. It binds only to muscarinic receptors. e. It is produced by cholinergic neurons.

e

382. Sharks, sea snakes, drunk Aussies... You've managed to avoid them all during your trip to Queensland. Too bad nobody warned you about box jellies. Their venom works by assembling porins, which within seconds begin to poke holes in the erythrocytes of the unfortunate victim. If your heart is still beating by the time you make it to the hospital, what treatment would be most effective in saving your life? a. Calcium injections because your ECG shows smaller than normal T-waves, indicating slower repolarization of the cardiac action potential caused by the rapid influx of sodium ions. b. Calcium injections because your ECG shows larger than normal T-waves, indicating faster repolarization of the pacemaker action potential caused by the rapid influx of calcium ions. c. Calcitonin injections because your ECG shows larger than normal T waves, indicating faster repolarization of the cardiac action potential caused by the rapid efflux of potassium ions. d. Calcitonin injections because your ECG shows smaller than normal T-waves, indicating slower depolarization of the pacemaker action potential caused by the rapid influx of calcium. e. Calcium injections because your ECG shows larger than normal T-waves indicating faster repolarization of the cardiac action potential caused by the rapid efflux of potassium ions.

e

383. Which of the following is true regarding the function of Thyroid Hormone? (Did not include an answer) a. T3 and T4 cause a decrease in the breakdown of nutrients b. T3 and T4 initiate the transcription of genes involved in glucose oxidation c. T3 and T4 decrease lipolysis, fetal development, and childhood tissue growth d. T3 and T4 decrease the body's sensitivity to catecholamines from the adrenal medulla by upregulation of receptors in the blood vessels e. T3 and T4 are critical for development of the brain in early childhood and support neurological function in adults

e


Conjuntos de estudio relacionados

CEH#18 - Oriyano - Cloud Technologies and Security

View Set

b 403, 405 colores (TRANSLATE SENTENCES & QUESTIONS)

View Set

Chapter 3: Organizational Change

View Set

Exam 2 review Legal 10, 11, 12, 13, 15, 16

View Set